Sunteți pe pagina 1din 80

TROPMED 2013

5. Pasien mengalami demam tinggi pada


TROPMED hari ke 4 dan didiagnosis malaria,
termasuk malaria golongan apa? Malaria
1. Patient brought to clinic because of some Malariae -> gara” quartana= malariae:
nodules on his arm and body and vivax and ovale= tertiana; subtertiana=
sometimes feel pain with pressure. On falciparum (demamnya continuous)
examinations we found erythematous
nodules, smooth and shiny, diffuse 6. Kerja di salon poultry dan
infiltrate with ill define border, no ayamnya pada mati, ada gejala
fluctuation or erosion. There were pain influenza, apa virus yg
on his elbow with nerve enlargement, mungkin nyerang? AI
best stain for examination?
Leprosy= skin lesion (nodule, macula), Adenoviridae= bikin diare;
anestesia, claw hand foot, drop nerve corona=MERS and SARS; herpes
enlargement viridae= cytomegalo and varicella;
A. KOH = jamur LIAT CATETAN
B. Giemsa = bacteria DHARMAWAN!!!!(catetan virus)
C. Zn Stain = micobacterium (leprosy), Gastroenteritis= rotavirus,
TB norovirus, astrovirus, adenovirus
D
E a. Orthomyxoviridae
2. Metode pengambilan darah untuk orang 7. Zat apa yg digunakan utk membersihkan
infective endocarditis? 3 kali di lokasi feses burung? Dari WHO jawabanya
berbeda, atau 1 lokasi interval waktu 10 sodium hypochloride 5%
menit 3 kali A. Iodin
B. Etanol 70%
3. Orang kena malaria, dicek demam tinggi, C. Etanol 90%
parasitemia +++, diberikan artemisinin, 3
hari kemudian saat dicek masih ada 8. Best virus isolation utk kasus di atas?
demam dan parasitemia ++++, keadaan A. Blood
spt ini disebut apa? B. Nasopharyngeal aspirate = soalnya
adanya di lower respi tract; droplets
A. Early treatment failure ( dibawah 28 soalnya ini yang paling dalem
hari) C. Hand swab
B. Late treatment failure (diatas 28 hari) D. Anal swab
C. Late paracitemia E. Nasal swab
D. Early paracitemia
E. Late manifestation 9. Orang kena malaria, demam tinggi, kena
parasitemia 3,5%, kenapa terjadi kondisi spt
4. Orang datang ke klinik, demam tinggi dll ini? Merozoit dikeluarkan dari liver menuju
(malaria), manifestasi klinis pasien ini darah ; sel darah merah yang pecah akibat
terjadi pada saat nyamuk dlm stadium kluarnya merozoit.
apa?
10. 45yo man was seen in the dermaotlogy clinic
A. Tropozoit because of some nodules on his arm and body
B. Sporozoit and sometimes feel pain with pressure. On
C. Merozoit examination was found erythematous
D. Mikrogametosit nodules, smooth and shiny diffused infiltrate
E. Makrogametosit with ill define border, no flunctuation or
erosion. There were pain on his elbow with B. Chloroquine
nerve enlargement. Diagnosis? C. Mefloquine
a. yaws = treponema palidum pertenue, kalo D. ACT
syphillis= treponema palidum palidum,
bntuknya bs macem” kyk papiloma; non tender 14. G3P2A0 gejala anak chorioretinitis, cerebral
ulcer, samping”nya demarcated( kyk ancur calcification, hepatomegaly, splenomegaly,
gtu), ngeinfeksi kulit, tulang sendi; dibagi berat badan lahir rendah. Ibunya dalah suster
primary=ada single lesi= mother yaws-> di RS siloam, tidak pernah kontak dengan
disseminate-> msk ke secondary stage nodule kucong. Diagnosis?
non tender ulcer, secondary=daughter yaws, Rubella= malformation berat, lesi pd mata,
multiple and lbh kecil, latent= gejala hilang tp glaucoma, defect septum, tuli sensorineural,
bs relapse sebagai tertiary stg, tertiary= bone hepatosplenomegaly, microcephaly,
and joint and soft tissue deformities bisa perubahan tulang;toxo= smua gejala diatas
spontaneously heal abis 3-6 bulan cmv= chorioretinitis, microcephaly, iugr,
b. leprosy (ada nerve enlargement bagian elbow oligo/polihydramnion, hydrops non imun,
& lutut ascites janin, hydrocephalus
c. anthrax = respi problem, black eschar/
carbuncle like lesion a. congenital toxoplasmosis
d. cellulitis (jawaban 2012. Yakin neh nggak b. congenital CMV
ngaco???) c. congenital rubella
e. carbuncle = ada pus kluar (drainage), tmenya d. congenital herpes
furuncle, kalo furuncle Cuma 1 kluaran pusnya
tp carbuncle bnyk; furuncle bisa jadi carbuncle 15. a baby is born with a rash identical.
Questioning reveal that the mother had a febrile
11. A laboratory examination of a leprosy patient illnes during the second teimester of
after 4 months MDT therapy revealed pregnancy. Examination reveals diffuse raised
Bacterial index 6+ and morphological index purple skin lesions. There is no pallor, jaundice,
50%. What is your conclution for this result ? or cyanosis. The eyes are cataract, the heart has a
Mulit gara” bacterial index >50%, 25-75% 3/6 systolic murmur and there is enlargement of
morphological indexnya; pausibacillar= 1-5 both the liver and spleen. There is no
nodular, morphological may not be acessible; lymphadenopathy. Of the following, the most
udh treatment masih gak sembuh-> resistent likely diagnosis is?

a. Pausibacillar leprosy patient sensitive to a. congenital toxoplasmosis = calcification


treatment cerebral, hydrocephalus, chorioretinitis
b. Multibacillar leprosy patient sensitive to b. congenital rubella = purple skin fix rubella,
treatment c. congenital CMV =IUGR, hepatosplenomegaly
c. Multibacillar leprosy patient resistent to d. congenital malaria
treatment e. congenital varicella
d. Pausibacillar leprosy patient resistent to
treatment 16 A baby is born with a rash identical. History
e. Pausibacillar leprosy patient prone to nerve taking reveals that the mother had a febrile illness
damage during the second trimester of pregnancy.
Examination reveals diffuse raised purple skin
12. Yang harus di considerate untuk pasien lesions. There is no pallor, jaundice, or cyanosis.
suspect leprosy? Lesi di tubuh yg painless & The baby has spread over his body especially on
numb arm and legs also fever. What is the most likely
diagnosis ?
13. Pasien kena malaria, best drug of choice:
a. Celulitis =necrotizing celulitis ->paling
A. Doxycicline memungkinkan gara” yang lain gak
cocok gejalanya. Biasa gara” umbilical ptirus pubis, lay eggs juga on or near
infections tp celulitisnya Cuma di seams of clothes
abdominal doang biasanya. b. Scabies = biasa di sela”, ada excoriation (
b. Furuncle = gak krn cma superficial, slalu bekas garukan), webbed space of fingers,
ada pus pkoknya ngelay eggs di antara kulit mati
c. Erythema nodosum = biasa di orgtua makanya warnanya putih
d. Multibacillar leprosy = jarang congenital c. Tinea corporis = jamur sluruh badan
and symptoms gk kyk gni d. Dermatitis atopik = autoimmune, ada
e. Pausibacillar leprosy= jarang congenital pemicu
and symptoms gk kyk gni e. Psoriasis = autoimmune,
hyperproliferation of skin keratin
17 Residen mengeluh karena di ruangannya AC
terlalu dingin, suara berisik dan lampu terlalu 20 A patient on MDT treatment complaining
silau. Termasuk hazard kategori apa? Physical about red urine that he experinced on the first day
hazard of treatment. What is the suggestion for this
condition ?obat rifampicin
18 A 10-year-old boy who has moved to your
practice recently has sore throats, get high a. Stop Clofazimin, can cause pink
temperature. The child reports a runny nose, brownish discolouration
mild cough, and abdominal pain. Findings on b. Continue the medication, its not a drug
physical examination include a temperature of adverse effect
100.3°F (38°C), and vesicular lesions on the c. Stop the medication because of
soft palate. There is no cervical adenopathy or nefrotoxicity
rash. Of the following, the MOST likely d. Stop Dapsone can cause anemia
diagnosis is herpangima/ mouth blister= grey pin hemolytic
head size vesicle rupture to form large, fibrin e. Change the treatment to single drug
covered ulcer., karena coxsackie virus, echovirus, therapy
enterovirus, parechovirus, herpes simplex virus
and adenovirus tp plg sering coxsackie 21. A 27-year-old female has just returned
from a trip to Southeast Asia. In the past
a. Adenovirus infection = diare”, influenza 24 hours, she has developed shaking,
b. Coxsackie virus infection=prodrome: chills, and a temperature 40o C. A blood
fever, sore throat, dysphagia 1-2 hari smear reveals Plasmodium vivax. Which
preceeded exanthema, abdominal pain, of the following agents should be used to
oral lesion, headache, vomiting eradicate the extraerythrocytic phase of
c. Mononucleosis = the organism?
d. Sinusitis
e. Streptococcal pharyngitis = a. Primaquine = 1”nya obat yang bisa
ilangin merozoit
A 2 years old boy brought to dermatovenereology b. Pyrimethamine
clinic because of itching all of the body. He c. Quinacrine
complains itching on palm, side of finger, d. Chloroquine
buttock, and abdomen. Clinical manifestations e. Chologuanide
are papule, vesicle, pustule, erotion, and
excoriation especially at finger web, arm, 22. Ibu hamil mau bepergian ke Papua, obat
abdomen, side of hand and feet, itchy prophylaxis yg cocok buat dia?
especially at night.His neighbours have same Mefloquine ( tapi kata dokter sisir pake
problem.He usually play together with his kelambu)
neighbours.What is the diagnose this patients?
(nggak boleh dikasih doxi & kalo dikasih
a. Pediculosis corporis = gara” kutu debu chloroquin percuma krn mungkin
resisten) B. Hematocrite
C. IgM=primary
infecction baru kluar
hari ke 14=IgG=
23. A 26-year-old nulliparous woman, in her secondary hari k2 udh
seventh month of pregnancy complains kluar; IgM= hari k 3-5
of a 7 days history of fever especially at ilangnya 60-90 hari;
night. As she recall, 2 weeks ago she ate
gado-gado in coastal area when she out
for duty for several days. She also had
diarrhea and abdominal discomfort. On 28 Workplace health hazard can cause
physical examination reveals blood reaction. Asthma, and dermatitis caysed
pressure 110/70 mmHg, heart rate 60 by whicch reaction?
×/min, RR 20x/min, temp. 39.1°C.
Tongue is coated and tremor. Fetal USG a. immediate reaction-> alergi, hipersensitivitas;
shows normal. 4 tipe: 1= IgE( asthma and anaphylactic),2=
IgG, IgM ( drug induced hemolytic anemia),
What is the most likely etiologic factor 3= circulating antigen antibody immune
above? complex with complement ( serum sickness),
4= delayed hypersensitivity reaction, mediated
a. Salmonella typhi by t cell ( contact dermatitis dari poison
b. Plasmodium falciparum ivy,etc)-> delayed
c. Vibrio cholera b. acute ->
d. Hepatitis A c. gradual
e. Escherichia coli d. delayed -> t cell; mononucleocyte; 48-72 jem
abis exposure; type 4( delayed
24. Best laboratory examination to detect hypersensitivity)
S.typhii? Blood smear e. no reaction

25. Best treatment for scabies? Gamexane= 30 cowo aktif sexual partner tapi pake pelindung,
obat buat scabies and pediculocyte terus mau screening. Screening pake apa?
(scabimite= permethrin 5%)
a. HIV-RNA-> confirm diagnosis( lebih untuk
26. Tukang kebon dtg ke klinik dgn keluhan epid studies)
demam tinggi sampe 39,5 derajat, b. CD4 count -> untuk nentuin level severity HIV
profuse sweating, mual muntah, best lab c. Rapid test -> untuk screening
examination? (suspect malaria) Thick and d. western blot -> confirm diagnosis
thin blood smear e. HIV-ELISA -> confirm diagnosis

31 Elephantiasis is caused by the obstruction of :


A Blood culture a. The arteries by microfilariae
B Stool b. The arteries by adult worm
C Saliva c. The lymphatics by microfilariae -> biasa
D di darah
E d. The chronic lymphatics by adult worm->
di lymphatics
27 Transmisi penyakit yg mungkin utk kasus e. Fibrosis of the lymphatic
diatas? Direct skin scale contact (digigit nyamuk)
32 A 40-year-old man came to the clinic with
Anak demam 2 hari, dokternya suspek DBD. complaints of having fever more than one-week,
mau cek apa? nausea, and fatigue. He came back from Timika,
A. NS= <1 mggu (1-8) Papua. On blood smear examination, there are
late trophozoites forms, schizonts with 12-24 b. Air borne
merozoites, and the infected red cell is c. Fecal-oral (suspect polio)
enlarged.Diagnosis?vivax sama ovale membesar d. Close contact
makanya dibedain lewat jumlah schizontnya P e. Animal bite
vivax-> merozoit eritrosit 12-18, ovale=8-10;
malaria=8; falciparum=8-24 35 A 5 year old boy was brought to emergency
department with complaining of paresthesia of
33 A 10-month-old healthy male infant the fingers and lower extremity muscle weakness
travelling with his family to Africa was since two days ago. The weakness progress
exposed 4 days ago to his 4-year-old native involves the truncal muscles. Possible
African cousin who was ill at the time with diagnsosis?
fever, cough, coryza and conjunctivitis. The
baby was at risk to get infected by: a. Guillain-Barré syndrome (ascending
paralysis) -> biasa dari campilobacter
*morbillivirus= measles/ rubeolla= 3C, lbh jejuni(->diare, GI syndrome), cmv,
bsar drpd rubella, dari leher k sluruh tubuh; ebv, hiv/ bacterial infection kyk
togavirus= rubella/ campak jerman-> demam campylobacter; trus ada infeksi
lbh ringan, ada lymphadenopathy baru rash sblmnya
merah” lbh kcl drpd measles dari kepala k b. Poliomyelitis-> lgsg, akut, cuma lower
badan; scarlet fever= strawberry tongue, rash extremities (biasanya unilateral)
di daerah lipatan, (+) blanching test; varicella= c. Encephalitis
varicella zoster, reactivation-> herpes zoster, d. Meningitis
dari muka ke badan, vesiclenya mix (ada yg e. Multiple Sclerosis -> muncul dari mata
baru and old), nular dari sblm vesicle muncul; (optic neuritis), weakness, tingling,
hand foot and mouth disease= coxsackie and numbness, blurred vision
enterovirus, di kaki mulut and tangan, gejala
ringan berupa vesicle-> herpangina trmsuk 36 A 53-year-old male farmer developed low-
HFMD; roseola= demam hilang rash muncul grade fever, abdominal pain, and diarrhea. He
LIAT TABEL DHARMAWAN! has a skin lesion of black eschar surrounded
by vesicle and edema near his mouth. A week
a. Corynebacterium diphtheriae. before, his notice many cattle in his farm were
b. Haemophilus influenzae sick, but he still eats a half-done beef steak
c. Bordetella pertussis from his cattle.
d. Morbilivirus  measles
e. Coxsackievirus What is the following microorganism is most
likely involved in this case?
34 A 65-year-old-woman suddenly had a. Bacteroides fragillis
paralysis of her left leg. A few days ago, she b. Leptospira interrogans
had headache, fever, sore throat, and nausea. c. Bacillus anthracis (bakteri gram
She lives with her son; daughter in law and a positif)
young grandchild who’s just received a routine d. Pasteurella pestis
oral vaccination. She is taking e. Borrelia burgdorferi
immunosuppressant for her kidney’s
transplant. Her vital signs and cranial nerve 37 cowo, MSM (male to males sex),
examination are normal. A head CT scan and datang ke poliklinik mau evaluasi
lumbar MRI are also normal. -> kemungkinan treatment HIV nya. 1 minggu lalu,
polio ( fecal-oral) HIV + dan CD4 <350. Next step
management?
What is the most likely transmission of her
-Kalo ada secondary infection gak boleh start
infection?
a. Droplet arv, takutnya ada reaksi imunnya lgsg tinggi-> 1-
2 mggu baru mulai-> kalo gak bisa ada gejala gastroenteritis while visiting small
sepsis; cd4 <350 baru kasih antiretroviral, >500 town in Indonesia and tried some
harapan hidup sama kyk org normal Indonesian traditional food. The onset
of the disease is abrupt with
abdominal cramps and watery
a. start arv diarrhea. She had no fever or nausea
b. cek tb or vomiting. The symptoms have
c. kasi prophylaxis resolved within 24 hour and no
d. rapid test subsequent recurrences. They report
e. do nothing the disease to district public health
office. The investigation found that one
38 52- year old man is brought to the of the food products eaten by this
emergency department in an unconscious tourist was contaminated by suspected
condition. On examination, he was death form pathogens. What is the suspected
more than one hour. On autopsy of the brain, pathogen may cause the disease above?
there is cyst‘s fluid containing a. Salmonella typhi -> ada fever, abdominal
protoscoleces( mature cyst) with hooklets and cramps, traveller’s diarrhea di India
many daughter cysts were found. What is the b. Shigella dysenteriae -> berdarah
most likely diagnosis of this disease? c. Enterohemorragic E. coli -> berdarah
d. Staphylococcus aureus
a. Hydatidosis -> soalnya ada daughter e. Enterotoxigenic E.coli -> traveller’s diarrhea;
cysts, cystnya bisa pecah-> shock bisa giardia lambia, cholera, rotavirus( biasa
b. Cysticercosis -> gak ada daughter cyst, di anak kecil), norovirus( di cruise ship
bukan cuma neuro tp bs k otot juga biasanya)
c. Cerebral malaria
d. Hstoplasmosis 41 A 3-year-old boy was brought to emergency
e. Toxoplasmosis unit in Tangerang Hospital with stridor,
dyspnea, and “croupy” cough ( ciri” pertussis-
39 A 27 year old women suffering fever for 2 > whooping cough), What is the most likely
day. She brought to emergency department pathogenic agent that causes the patient’s
unconsciously and she had general seizure 2 hour disease? Stridor-> gara” ada pseudomembrane
ago. On physical examination she is comatous, ‘jatoh’
GCS (Glasgow Coma Scale) : 3; BP 90/60 a. Bordetella pertussis-> whooping cough
mmHg; t 41 °C. Lab. finding: Hb 10, 3 gr%; bukan croupy
WBC 5800/mm3; platelet 39.000/mm3; there is b. Haemophilus influenzae
schizon form on blood smear. Why the c. Coxsackievirus
pathogeniv agent of this disease becoming the d. B. Parapertussis
most dangerous species? e. C. diphtheriae -> croupy cough,
pseudomembrane bikin difteri toxin->
a. It make long term relaps hematogenous jd nyebar-> yg bkin
b. Intermittent fever bahaya
c. Gametocyte formation take place in
visceral organ 42 What is the laboratory test to confirm the
d. Can make capillary obstruction in Central diagnosis in this patient?
Nervous System by parasite contain
erythrocyte = PFRB1 receptor untuk a. Blood culture
masuk ke erythrocyte b. Throat swab culture->definitive diagnosis
e. There are many drug resistance against P. c. Chest X-ray
falciparum d. Measurement of IgG antibody serum
e. Urine culture
40 A female tourist developed
43 A 70 years old women, come to w/ nadi gak keukur)
dermatovenereology clinic because she
complains pain, edema on her left eyelid and 46 49 yo man come to dermatologist because of
there is some vesicle in her left forehead. She creeping in left elbow on day 3 trip. Eruption
complain pain on her forehead 2 days ago, but 6cm, linear, erythematobolbulus. Most possible
vesicle come out 1 day ago.She sometimes fells causees?
headache and fever. Clinical manifestation are
grouped vesicle, eritema base at her left a. dermacentor andersoni
face.What is the diagnose of this patient? b. lactrodectus mancans
c. loxoxsceles laeta
a. Varicella-> latent kalo teraktivasi jadi d. lytta sesuatu
zoster; gejala gak sampe parah bngt e. paederus peregrinus (tomcat) -> bikin
b. Herpes zoster -> gejalanya parah sampe erythematobolbulus
edem, cenderung unilateral
c. Herpes simpleks -> bilateral, bisa mulai 47 A 44-year old man on a business trip to South
dari oral Kalimantan from Jakarta presented to the
d. Dermatitis venenata emergency department with symptoms of
e. Dermatitis iritan diarrhea and abdominal pain. On physical
examination was found pitting edema of the
44 A 19-year-old woman comes to abdominal wall and of the legs. A stool
Emergency Department with signs and specimen was sent to the laboratory for culture
symptoms of poisoning of an agent include and for ova and parasites. Blood was drawn for a
nausea, vomiting, abdominal cramps, CBC. The culture was negative for enteric
diarrhea, excessive salivation, headache, pathogens. The patient was found to have
giddiness, rhinorrhea, tightness in chest, eosinophilia. The wet mounts from the
pin-point pupils  parasimpatetik activity, concentrated stool specimens revealed large,
mental confusion, and muscle twitching. ellipsoidal, operculated parasite eggs with thin,
Which of the following is the most likely transparent shells (140 x 85 µm).
agent caused those symptoms?
Of the following, which parasite can be the
a. Acetaminophen cause of this disorder :
b. Barbiturate a. Schistosoma japonicum-> vena
mesenterica superior
c. Carbamate b. Fasciolopsis buski -> di hati bikin
d. Cyanide hepatosplenomegaly
e. Opiate-> mustinya no chest tightness, c. Trichinella spiralis -> di otot
melalui opioid receptor, reduced GI d. Echinococcus granulosus->
activity, hidatidosis
e. Clonorchis sinensis-> makan ikan
45 Orang demam tinggi, nyeri belakang mata, mentah
demam, Hb Ht naek, berdasarkan WHO, 48 hematuria dan oval shaped and ada di terminal
diagnosisnya apa? spine?
schistosoma hematobium-> di vena plexus
A. Dengue fever -> derajat 0 hemoroidalis ( vena bladder and rectum-> bikin
B. DHF (hemokonsentrasi & tanda hematuria)
perdarahan) -> soalnya Hb Ht naik-> plasma
leakage; DBD derajat 1( tourniquet test 49 40yo berenang di danau lindu, gatal2, besok
+)&2( pendarahan spontan, ) muncul papul, most likely pathogen?
C. Severe Dengue -> DSS-> derajat 3 (gagal schistosoma japonicum-> telur di feces-> telur
circulation, nadi and BP turun, mulai altered pecah di danau-> pecah jadi miracidium->
mental status, mulai gelisah)&4( shock berat sporokista 1-> sporokista 2-> serkaria-
>serkarianya brenang bebas di danau nembus larvae
kulit-> larva and dewasa di intrahepatic blood-> c. examine the blood smear for the presence
ke vein-> kembang biak disana of adults worm
d. determine the titer of IgE antibody
50 Bentuk infeksi yg nyerang pasien tsb (soal against the parasites
49): larva filariform e. examine the blood smear for the presence
of microfilaria
51 Manifestasi klinisnya: (dari buku UI:
stadium 1 urtikaria, kalo intoksikasi 53 Treatment yg cocok utk case diatas (suspect
disertai demam, hepatomegaly & filariasis)? DEC (Diethylcarbamazine Citrate)
eosinophilia tinggi ; stadium 2 sindrom combination with albendazol=>wkt kt bljr
disentri ; stadium 3 sirosis hati & sempet gw sebut ini
splenomegaly)->gejalanya schistosoma
japonicum, swimmer itch, complication: 54 A 40 year old man came to hospital with four
*Toxocara sp. Ascarit dari anjing(canis) sama times generalized tonic-clonic seizure since two
kucing(cati), bisa bkin visceral larva migrans, week ago. There was no history of seizure and
nyerang anak kecil yg suka eating dirt, symptoms head trauma. A one year ago, he had business in
seizure, encephalitis, myocarditis, hepatomegaly, Papua for six months. During his stay in there, he
pruritic rash, urtikaria, preorbital edema, often consumed under-cooked pork. Brain MRI
showed a small cystic with invaginated scolex
a. creeping eruption =associated with cutaneous inside. What of this following is the most likely
larva migrans; filariform larva (hookworms)-> inside cyst?
strongyloides stercoralis, necator americanus,
ancylostoma duodenale a. Taenia eggs
b. billiary cirrhosis (gw jawab ini) b. Cycticercosis cellulose =larvanya solium
c. cholecystitis (jawaban 2012, yakin nih?) -> c. Cysticercosis bovis = larvanya saginata
karena eggnya ditmukan di gall bladder, d. Taenia solium worm
associated with inflammatory granulomatous e. Taenia saginata worm
infection
d. pruruitus ani = entrobius vermicularis( cacing 55 Possible etiology case di atas? Undercooked
kremi) pork
e. visceral larva migran= toxocara sp
56. a 39 yo woman whose husband has a pig at
52 a 23yo man presents with extreme swelling of home kept for house purposes come to the family
his legs and scrotum. The skin associated with doctor with symptoms of periorbital edema,
the swollen areas is thick and scally. The patient weakness and muscle pain. What is the most
admits to an episode of fever associated with probable parasite causative in this case?
enlargement of inguinal lymph nodes some times
ago, but did not think much of it. Which of the a. schistosoma japonicum = dari snail karena
following is the best method to make diagnosis? masuk airnya jadi nimbus kulit
*pembesaran scrotum, payudara, full extremities b. f. buski = hepatomegaly, sheep liver fluke
Cuma di wucheria bancrofti, ada juga hiluria, (reservoir di domba), transmisi dari
nocturnal makanya ceknya harus jem 9 malem metacercaria yang ada di watercress plant,
symptom nya biliary colic, jaundice,
ampe 3 pagi, cacing dewasa di lymph node, kalo
generalized abdominal pain, cholecystitis,
microfilaria di blood maknya ceknya cholelithiasis.
microfilarianya; brugia timori and malayi di c. trichinella spiralis = dari pork atau wild
distal part of extremities animals, subconjunctivae hemorrhage
d. echinococcus granulosus = hidatidosis, dari
a. examine the stool for presence of eggs dog tape worm. Membentuk kista yang dapat
b. examine the stool for the presence of menyebabkan anafilaksis shock.
e. clonorchis sinesis=ikan mentah, ke hati dan 60 Cara ngambil specimen darah yg baik?
kantong empedu. Gw lupa pilihannya, pokoknya di
handout dia blg: diambil saat yg tepat
57 Etiologic cause case di atas? (cthnya pas suhu naek), peralatan yg
a. eating larvae in contaminated food baik, ambil pas pasien blm minum
b. eating eggs in contaminated food antibiotic
c. eating cyst in contaminated food
d. undercooked pork 61 56 year old sheep farmer presents with
e. undercooked beef abdominal pain. Liver is enlarged with
palpable mass at right upper quadrant.
58 A patient complains of having nail size CT scan reveals a large encapsulated
excrement from his anus. On the stool lesion consistent with HYDATID
examination, parasite eggs contains a CYST. What organism may have cased
hexacanth embryo with six hooklets this? Echinococcus granulosus
surrounded by radially striated spherical
shell, 30 to 40 u in diameter were found, the 62 manifestasi apa yang paling berbahaya
most likely diagnosis of this patient is : ketika surgucal removal case di atas?
bayangin aj, ni soal di angkatan lu
a. Saginata taeniasis (di feses biasa sempet keluar tp ga ada yg nyatet
ditemuin proglotid) yang aktif atasnya. Anjeng bener emg…!!!!!!!!!!!!!
b. Solium taeniasis yang pasif
c. Cysticercosis a. seizure
d. Oxyuriasis b. hemorrhage
e. Ascariasis c. peritonitis
d. anaphylactic shock
59 a 14 yo boy fell from bicycle, e. cardiac arrest
last immunization at 18month,
(4th doses) 4-6 year
immunization wasn’t done. 63 The patient was 3 y.o girl who was
What terapy should be given? seen by her pediatrician for a routine
Tetanus Ig atau antitetanus physical exam. Her mother was
serum tp hrs cek concerned about her daughter’s poor
hypersensitivitas dulu sblm appetite. Physical exam revealed that
dksh full dose the child was small for her age and had
a slightly enlarged liver. Blood was
collected for a routine CBC, since she
had previously been slightly anemic.
Hb normal, eosinophilia, no travel
history, melihara anak anjing di rumah.
Possible cause? Toxocara canis

64 Manifestasi klinis case di atas? Visceral


larva migrans

a. tetanus Ig
b. human iv Ig 65 A 40-year-old man came to the clinic
c. tetanus toxoid with complaints of having fever more
d. anti tetanic serum than one-week, nausea, and fatigue. He
came back from Timika, Papua. On
blood smear examination, there are late
trophozoites forms, schizonts with 12-
24 merozoites, and the infected red the family live in village, tempoarte climate.
cell is enlarged.Diagnosis? P. vivax Previous 10 days, he worked on irrigation
channel and isinya air sungai. What is your
66. Cewe, flank pain, pas urinalysis hasilnya diagnosis?
positive nitrate dan many leucoytes, next
step> a. treponemasis
a. urine culture= curiga UTI jd
cek jumlah bakteri& penyebab b. shigelosis
b. PCT( paracetamol) oral c. salmonelasis
c. USG d.
d. . e. leptospirosis =icteri/anicteric, renal failure, ke
e. IV ceftriaxone brain, uveitis, subconjuctiva hemorrhage,
67. 9 years old susah nafas, noisy breathing muscle pain biasanya gastrocnemius
sejak 3 hari yang lalum juga ada fever cough
myalgia headache hoarseness. Refuse to eat 70 A 35-year old man working on pig farm is
karena sakit tenggorokan, ada pembesaran brought by his employer to the emergency
KGB leher (bull neck appearance, bisa
gara”edem juga)), white membrane di department of Siloam hospital because of seizure
tonsil-> ciri”difteri, terbentuk and bilateral lower extremity weakness. CT scan
pseudomembrane. Additional exam? of the head reveals several calcified regions.
f. Neck xray to reveal laryngeal Laboratory examinations show WBC count of
edema
g. CBC – liat infeksi bakteri 10.800/mm3, with eosinophils. Brain tissue
h. Throab swab – untuk gram biopsy reveals scolex with hooklets. Which one
staining danswab kultur is the treatment of choice for this case?
i. . Neurocysticsercosis
j. refer THT

68 A boy brought by his mother to the a. Sulfadiazine-pirimetamin = obat malaria


emergency department of Siloam hospital b. Artemisinin =obat malaria
because of vomit & dyspnoea. According to his c. Albendazol= lbh buat neurosistiserkosis;
mother, her son was playing in the backyard. On praziquantel= 2nd choice,di jaringan lain
physical examination, the boy was found to have and albendazol combination
high fever (39 ºC) and multiple stinging on the d. Metronidazol
back. The most likely species of insect bites is : e. Itrakonazol

a. Hymenoptera sp = bee 71 A 35-year-old man as a farm worker who


b. Dermacentos andersoni was working with pesticides is brought to the
c. Lactodectus mactan = black widow emergency room with headache, vomiting,
d. Loxoxceles laeta =spider salivation, diarrhea, muscle fasciculation,
e. Lytta vesicatoria= kumbang difficulty walking, and difficulty speaking. His
*kumbang paederus= tomcat-> erythematous clothing has been removed, he has been
washed, and he has been given activated
bulous lesion, sudden onset
charcoal. What is the most effective remaining
treatment for this case of pesticide poisoning?

a. Epinephrine
69 21yo man come to emergency department b. Antacid
due to high fever, headache, nausea and c. Spironolactone
vomiting and constant sleepiness for 5 days. At d. Atropine = anticholinergic soalnya dia
admission, he look lethargic and dehydrated. keracunan cholinergic berlebihan;
At PF, he look jaundice, conjuctival injection pralidoxine gak boleh dikasih sendiri
and … there are liver dysfunction and soalnya dia nicotinic harus dipake
increased creatinine. The relative stated that atropine
e. Hidrochlorothiazide
d. colonization
72. Occcult filariasis is Lymphatic filariasis e. genetic
based upon
*occult filariasis= bnyk eosinophilia mkanya
ada gejala asthma, bnyk microfilia tp gak ada 73. frustated patient diagnosed with leprosy.
gejala kecuali yg kyk asthma gtu-> TPE
(tropical pulmonary eosinophilia) He is afraid that he is cursed abd going to die.
a. infection As a medical doctor, what will you say to the
b. allergy= karena IgE patient?
c. infestation

dan teratur, perbandinga kepala ekor 2:1,


a. rest all day and don’t do his job parasit apa?
b. cannot be cured
c. treatment in leprocy house a. wuchereria bancrofti= ruang kepala 1:1, lekuk
d. look out for wound badan halus , penyebaran inti merata, ekor tdk
e. highly contagious and contact his family = ada inti tambahan
terserah hati kalian!:) b. brugia malayi = 2:1, kaku, inti tumpeng tindih,
sarungnya ngambil warna giemsa
c. brugia timori = 3:1, kaku bngt, penyebaran inti
74 a patient come with an itchy recured fungal tumpang tindih
infection on feet, doctor already give him d. onchocerca volvulus
some medication for fungal infection, what e. loa loa = bikin filariasis subkutan
can be advised to make him healthy?
*lucio phenomenon= atrophy sampe necrosis di 78 One member of the NGO (Non-Governmental
leprosy, jadi item”gtu Organization) team from Indonesia has been
reported dead in Mexico because of a spider’s
a. don’t eat seafood bite. The possible cause of this disorder is ;
b. always use sandals
c. keep the skin dry and clean a. Erucism = gara”caterpillar
d, don’t use shoes all day long b. Lepidopterism = gara” adult butterfly
e. don’t scratch, it will spread the infection c. Tick paralysis = gara”kutu anjing
d. Arachnidism
75 Komplikasi demam thypoid paling sering e. Delusional parasitosis = gangguan
dimana? Ileum distal-> peyer’s patch, bkin sikologis dimana pasien ngira dia ada
perforasi (pilihannya wkt itu ada colon gt2) pruritus gara”parasite, pengguna
metamphetamine
76 Cewe kena tusuk jarum suntik orang yg kena
HIV, langkah selanjutnya?
79 Orang HIV, ada riwayat TB dan Hepatitis
A. Rapid test B. kombinasi obat yg dipake? Gw lupa, tp gw
B. Post-exposure HIV prophylaxis (gw jawabnya yg Emicitabine, Tenovovir,
jawab ini) -> zenovudin atau tenovovir + Efavirenz. Pelajarin sendiri aja ye? Lamivudine
lamivudine atau empirisitabin selama 4 + zidovudine +evafirenze; kalo gak tb eva
wks , initiation secepat hari, lbh diganti nefirapine
dianjurkan dilakukan <24 jem or else risk
lbh tinggi 80 a 45 yo women, ke rs karena diare 3 hari,
C. Start antiviral tiap hari 5-6x, sebelumnya dirawat di rs karena
D. Do nothing pneumonia. Pernah ada history nasopharunx
CA, faktor yg mempengaruhi diare?
77 cewe dtg dgn keluhan bengkak di tungkai
bawah, thick blood smear ada microfilaria a. agent not usually cause disease in healthy
ciri deeply stain giemsa, inti sampe ke ekor person
b. evolution of pathogen shg bisa ke new host e?
c. development of drug resistant of pathogen
d. non-adherence pasien dlm minum penumonia
drugs
e. pake counterfeit drugs 3. lab exam buat salmonella?

a. IgM salmonella = awal


81. Anak sekolah diperiksa ada gatel2 dll (gejala
scabies), trus temen2nya di kelas jg ngalami b. anti HCV
keluhan yg sama. Langkah yg seharusnya
dilakukan?

A Preventif 7. which subtype are known to cause HPAI?


B Kuratif
a. H1 and H3
C Rehabilitatif
D Promotif b. H2 and H4
E Environmental modification (gw nembak
jawab ini) = karena scabies sgt sensitive terhdp c. H5 and H7
environment
d. H6 dan H8

TROPMED 2012 e. H9 and H11

(thio) Kerja di poultry dan ayamnya pada mati.


35 tahun, ada gejala influenza, apa virusnya?
8. hep B positive, HIV positive, tb positive,
a. orthomyxoviridae treatment untuk HIV? TEE”

(dion) anak luka trus uda di ksi vaksin tetauns,


12. dari kasus 11, manifestasi apa yang paling
tapi hanya sekali. Management skrg apa?
berbahaya ketika surgucal removal?
Jawaban : Tig
a. seizure

b. hemorrhage
(guntur) 24 thn skoleks ditemukan di CT scan
c. peritonitis
atau MRI.
d. anaphylactic shock
a. taenia solium
b. b. toxo e. cardiac arrest
c. c. echinococcus

2. salmonella typhii, lab diagnosis? 14. manifestation of infection above:


*widal= spesifisitas gak tinggi
a.stool culture= + week ke 3 ilang week 8, di a. creeping eruption
carrier juga ada
b. biliary cirrhosis
b. blood culture= gold std
c. cysticercosis-> mustinya cholesistitis
c. swab
d. pruritus ani
d?
e. visceral larva migrant

21. treatment no 20

15. a 39 yo woman whose husband has a pig at a. metronidszole


home kept for house purposes come to the family
doctor with symptoms of periorbital edema, b. dietilcarbamasin
weakness and muscle pain. What is the most
probable parasite causative in this case? c. mebendazole

a. schistosoma japonicum

b. f. buski 23. 49 yo man come to dermatologist because of


creeping in left elbow on day 3 trip. Eruption
c. trichinella spiralis 6cm, linear, erythematobolbulus. Most possible
causees?
d. echinococcus granulosus
a. dermacentor andersoni
e. clonorchis sinesis
b. lactrodectus mancans

c. loxoxsceles laeta
16. atasnya soal trichinella spiralis?
d. lytta sesuatu
a. eating larvae in contaminated food
e. paederus peregrinus= tomcat
b. eating eggs in contaminated food

c. eating cyst in contaminated food


25. cewe dtg dgn keluhan bengkak di tungkai
d. undercooked pork bawah, thick blood smear ada microfilaria ciri
deeply stain giemsa, inti sampe ke ekor, parasit
e. undercooked beef apa?

a. wuchereria bancrofti
17. ada org datag dengan demam lebih dari 1 b. brugia malayi
minggu. Dari blood smear ada parasit bentuk
cincin dgn pembesaran sel darah merah. Kena c. brugia timori
apa?
d. onchocerca volvulus
a. plasmodium vivax
e. loa loa
b. p. malariae

c. p. falciparum
26. frustated patient diagnosed with leprosy. He
d. p. ovale is afraid that he is cursed abd going to die. As a
medical doctor, what will you say to the patient?
e. p. knowlesi
a. rest all day and don’t do his job

b. cannot be cured
18. demam tiap hari ke-4
c. treatment in leprocy house
jaw : plasmodium malariae= quartana
d. look out for wound c. erythema nodosum

e. highly contagious and contact his family d. multibacillar leprosy

e. pausibacillar leprosy

28. a patient come with an itchy recured fungal


infection on feet, doctor already give him some
medication for fungal infection, what can be 33. patient on MDT treatment complaining about
advised to make him healthy? red urine that he experience on first dat of
treatment. Suggestion?rifampicin-> continue
a. don’t eat seafood medication

b. always use sandals a. stop clofazimin, can cause pink brownies


discoloration
c. keep the skin dry and clean
b. continue medication, it is not drug adverse
d, don’t use shoes all day long effect
e. don’t scratch, it will spread the infection c. stop the medication because of nephrotoxicity

d. stop dapsone can cause anemia hemolytic


29. a 45yo man was seen in the dermaotlogy e. change the treatment to single drug therapy
clinic because of some nodules on his arm and
bpdy and sometimes feel pain with pressure. On
examination was found erythematous nodules,
smooth and shiny diffused infiltrate with ill 35. upper quadrant pain dan fever. Liver
define border, no flunctuation or erosion. There enlargement and tenderness. Baru pulang dari
were pain on his elbow with nerve enlargement. thailand. Chill and sweat waktu fever. Terlihat
Diagnosis? 3.5% paracitemia. Explain mna yang benar?

a. yaws a. paracutemia … from tissue injury

b. leprosy b. paracitemia release of merozoit

c. anthrax c. paracitemia due to sporozoit initiate in human


infection
d. celullitis
d. paracitemua dri repeated cycle schizon from
e. carbuncle liver to erytocyte

31. a 21 yo man was seen in dermatology clinic 36. treatment malaria


because of sore and redness on his previous skin
lesion. He was on MDT treatment since 1.5 year a.
ago. On skin examination found some painful
erythematous nodule spread over his body b. chloroquinee HCl intramuscular
especially on arm and legs also fever. What is the c. artemeter + lumefanfrine
most likely diagnosis?
d. primaquiine
a. celullitis

b. furuncle
37. a 45 yo women, ke rs karena diare 3 hari, tiap d. mefloquine 2 days before
hari 5-6x, sebelumnya dirawat di rs karena
pneumonia. Pernah ada history nasopharunx CA, e. primaquine 2 weeks before
faktor yg mempengaruhi diare?

a. agent not usually cause disease in healthy


person 44. management soal no 43:

b. evolution of pathogen shg bisa ke new host a. hospitalisation with iv

c. development of drug resistant of pathogen b. outpatient meal soup

d. non-adherence pasien dlm minum penumonia c.


drugs d. jambu biji
e. pake counterfeit drugs e. domperidone

39. a 14 yo boy fell from bicycle, last 48. G3P2A0 gejala anak chorioretinitis, cerebral
immunization at 18month, (4th doses) 4-6 year calcification, hepatomegaly, splenomegaly, berat
immunization wasn’t done. What terapy should badan lahir rendah. Ibunya dalah suster di RS
be given? siloam, tidak pernah kontak dengan kucong.
a. tetanus Ig Diagnosis?

b. human iv Ig a. congenital toxoplasmosis

c. tetanus toxoid b. congenital CMV

d. anti tetanic serum c. congenital rubella

d. congenital herpes

41. 6yo girl was brought to clinic, cough,


vomiting. PE found: crackles in both lungs. 49. hematuria dan oval shaped and ada di
Treatment? oseltamivir terminal spine?

Jaw : schistosoma
42. 24yo woman datang ke dokter pribadi mau ke hematobium
papua selama 1 minggu. Ibunya uda menikah, 2
bulan gk haid, diksi prophylaxis apa? Pakenya
kelambu , kalo gak ada pake; kalo sbg obat= 50. a 19yo woman come to emergency deoarment
kina+ tetra/clinda/doxy +primaquin (2nd line), with sugn and sympotms of poisoning of an agent
first linenya ACT( artesunat, amodiaquine, include nausea, vomiting, abdominal cramp,
primaquin-> u/ falciparum; kalo yg lain sama tp diarrhea, excessive salivation, headache,
primaquinya dilanjutin sampe hari ke …..) giddiness, rhinorrhea, tighness in chesr, pin-point
pupil, mental confusion and muscle twitching.
a. sulfa- piremitamin 2 weeks before Which of the following is the most likely agent
b. an… caused those symptoms?

c. doxycycline 2 days before a. acetaminophen


b. barbiturate e. examine the blood smear for the presence
of microfilaria
c. carbamate

d. cyanide 59. cowo, MSM, datang ke poliklinik mau


evaluasi treatment HIV nya. 1 minggu lalu, HIV
e. opiate + dan CD4 <350. Next step management?

a. start arv
51. 40yo berenang di danau lindu, gatal2, besok b. cek tb
muncul papul, most likely pathogen?
c. kasi provilaksis cotrim
c. schistosoma japonicum
d. jgn kasi arv sampao cd4 <200

e. kasi prophylaxis clatromycin


53. soal no 51 ditanta bisa nyebapin apa?

a. creeping eruption
63. 2yo brought to dermatovenereology clinic
b. billiary cirrhosis because itching all over the body. He complained
c. cholecystitis of itching on palm, side of finger, buttock and
abdomen. Clinical manifestations are papule,
d. pruruitus ani vescicle, pustule, erotion and excoriation,
especially at finger web, arm, abdomen, side of
e. visceral larva migran hand and feet. Itchy especially at night. His
neighbour have the same problem. He usually
play together with his neighbour. What is the
diagnosis of this patient?
55. makan gado2, trus ever, lidah coated, termor.
Causative agent menyebabkan perforasi dimana? a. pediculosis corporis
a. ileum b. scabies

c. tinea corporis
57. a 23yo man presents with extreme swelling of d. dermatitis atopik
his legs and scrotum. The skin associated with the
swollen areas is thick and scally. The patient e. psoriasis
admits to an episode of fever associated with
enlargement of inguinal lymph nodes some times
ago, but did not think much of it.
64. 8 day old infant, any loud noise appears to
Which of the following is the best method to cause him pain, ada muscle tightening and back
make diagnosis? arching, causing the head to nearly touch his
feet. PF reveals only drued packing on his
a. examine the stool for presence of eggs umbilical cord as is the local custom. He appear
b. examine the stool for the presence of normal until stimulated by loud noise ot touch
larvae and then began to cry, stiffens and arches his
c. examine the blood smear for the presence back. The stiffness continues until he calm down.
of adults worm Most likely diagnosis:
d. determine the titer of IgE antibody
against the parasites a. bacterial meningitis
b. botulism 72. cewe 55 thn, mau ke makkah, dia uda 10
tahun gak kesna, pemeriksaan fisik normal, perlu
c. generalized seizure vaksin apa?
d. TETANUS a. salmonella typhii
e. viral encephalitis b. tetanus

c. hepatitis a
67. ibu bawa anak umur 18 bulan ke ER dgn d. hepatitis b
keluhan runny nose, conjuctivitis, diarrhea. PF:
fever 40C 3 hari, rash 2-3mm mulai dari muka. e. meningiococcal (arab2)
Most likely causes:

jawban: morbili-> measles


73. 21yo man come to emergency department due
to high fever, headache, nausea and vomiting and
constant sleepiness for 5 days. At admission, he
69. a baby is born with a rash identical. look lethargic and dehydrated. At PF, he look
Questioning reveal that the mother had a febrile jaundice, conjuctival injection and … there are
illnes during the second teimester of pregnancy. liver dysfunction and increased creatinine. The
Examination reveals diffuse raised purple skin relative stated that the family live in village,
( blueberry muffin) lesions. There is no pallor, tempoarte climate. Previous 10 days, he worked
jaundice, or cyanosis. The eyes are cataract, the on irrigation channel and isinya air sungai. What
heart has a 3/6 systolic murmur and there is is your diagnosis?
enlargement of both the liver and spleen. There is
no lymphadenopathy. Of the following, the most a. treponemasis
likely diagnosis is?
b. shigelosis
a. congenital toxoplasmosis
c. salmonelasis
b. congenital rubella
d.
c. congenital CMV
e. leptospirosis
d. congenital malaria

e. congenital varicella
75. cowo aktif sexual partner tapi pake pelindung,
terus mau screening. Screening pake apa?

71. ke india 6 hari lalu, foul smelling and a. HIV-RNA


slimmy diarea-> ETEC
b. CD4 count
a. giardia lamblia= Thailand, traveller’s diare,
gak berdarah, steatorrhea c. Rapid Test

b. vibrio cholera= daerah banjir, air cucian beras d. western blot

c. shigella=ada berdarah e. HIV-ELISA


77. 67 yo man with long lasting diabetes and C. organism culture isolation
abscess in pedis dextra. PF BP 120/80, HR
120bpm, RR 28, tempt 37,8C. Hb 8, WBC
21,000, platelet 109,000. Next step evaluation
and management? 81. workplace health hazard can cause reaction.
Asthma, and dermatitis caysed by whicch
a. administer normal saline bolus reaction?

b. obtain culture a. immediate reaction= 1 time event, kyk nausea,


SOB
c. transfusion red pack blood cells
b. acute
d. platelet transfusion
c. gradual= asthma, dermatitis, bisa kambuh
e. bone marrow biopsy
d. delayed”= long term exposure

e. no reaction
78. boy 12yo dtang ke klinik demam 4 hari yg
lalu, semam, mual, skait kepala, fatique,
tourniquet +, apa arti hasilnya?
85. nenek dtg kidney transplant tinggal dgn anak
a. increased capillary liability n cucu. Anak bayi baru vaksin. Nenek merasa
kaki lemah sebelah. Vaksin apa bisa prevensi
b. devreased platelet count penyakit ini?

c. increase hematocrit a. HiB

d. decrease platelet function b. orosabin-> Polio yg live attenuated, kalo yg


salk itu yg mati
e. DHF= lab: platelet<100k
c. meningiococcal

d. varicella
79. anak demam 2 hari, dokter suspek DBD, cek?
e. hep B
a. NS1= 1-2 hari dr nata; 1-8 IPD UI

b. hematocrite

c.IgM= 3-5 hari

80. 38yo woman fever 10 days, malam makin


parah, ada fatique, dizzines, stomach discomfort,
constipation, loss of apetite. Suhu 38C BP 100/70
pulse 70x. mild abdominal pain, tremor and
coated tongue. Main specific standart lab exam
for diagnosis?-> Typhoid

a. PCR

b. Widal
MCQ TROPICAL MEDICINE 2010 b. Blood
c. Saliva
1. An apparently run-down but alert 34-year-old d. Lung biopsy
woman comes to your office after 6 months e. Nasopharyngeal swab
spent as a teacher in a rural West Sumba. Her
chief complaints are frequent headaches, 4. A 16-year-old boy presents with an annoying
occasionally nausea and vomiting, and cough, which he has had for about 3 weeks.
periodic fever. To rule out your differential The illness started with a runny nose. On
diagnosis, a smear of finger-stick blood was examination he is completely normal, but he
done. Which of the following choices would exhibits several episodes of severe coughing.
fit your diagnosis based on your microscopic He has difficulty to take deep breath and
examination of the blood smear? usually ends up with vomiting. Of the
a. Schizonts in red cells with 8-12 progeny following, the MOST likely diagnosis is
b. Rounded gametocytes present a. Tonsilitis
c. Enlarged, somewhat misshapen in red b. Bronchiolitis
cells c. Pertusis
d. Large ovoid parasites in some of the red d. Laryngitis
cells e. Diphteria
e. Band-shaped tropozoites in infected red 5. A young man was admitted to hospital after
cells increasing left arm pain and paresthesia.
Several days ago, he was bitten by a dog. His
2. A 20-year-old woman came to the emergency symptoms increased and were accompanied
room with high fever from 5 days ago. She by hand spasm and sweating on the right side
also has headache and pain in retroorbita, of the face and trunk. This patient was
abdominal pain and bloody stool. On admitted to the hospital the day after
physical examination, she was in an agitated developing dysphagia, hydrophobia,
condition. Her BP 90/60 mmHg, heart rate hypersalivation, and disorientation.
110x/ minute, RR 20x/ minute and there were Which of the following pathologic feature is
purpura in her arm, hand and leg. Which of most likely found in dog’s brain?
the following examination is the most a. Giant cell
appropriate to establish the diagnosis? b. Limfosit plasma biru
a. Prick test c. Negri bodies
b. Dengue IgM-IgG d. Inclusion bodies
c. Complete blood test e. Clue cell
d. Widal test
e. Stool analysis Question 6-7 are linked to the following case:
A 32-year-old worker from a farm comes with a
3. A 32-year-old female developed a “flu-like” painless dermal papule on his right hand which
syndrome with high fever up to 400C, started to develop since five days ago. The
anorexia, headache, and myalgia. Four days worker says as the lesion getting bigger, and the
later, she become confused and agitated with skin becomes black . He has tender axillaries
difficult to breath. Chest x-ray showed lymph node enlargement.  anthrax
parenchymal infiltrate. One of her kids has
also these symptoms, and has already passed 6. What antibiotic should the doctor give to
away. In order to provide laboratory treat the worker?
conformation of the pathogenic agent, a a. Tetracycline
tissue culture was ordered. b. Aminoglycoside
Which of the following would be the best c. Ciprofloxacin
specimen for isolating the pathogenic agent d. Neomycin
responsible for this infection? e. Crystalline penicillin G
a. Stool
7. Which of the following virulence factors is were sick, but he still eats a half-done beef
most likely to be involved in the pathogenesis steak from his cattle.
of illness? What is the following microorganism is most
a. Exotoxin likely involved in this case?
b. Endotoxin a. Bacteroides fragillis
c. α-hemolysin b. Leptospira interrogans
d. Lipopolysaccharide c. Bacillus anthracis
e. Antiphagotic factors d. Pasteurella pestis
e. Borrelia burgdorferi
8. A 45-year-old man was seen in the
dermatology clinic because of some nodules 11. A woman, recently returned from Lampung,
on his arm and body and sometimes feel pain complains of having paroxysmal attacks of
with pressure. On examinations we found chills, fever, and sweating; these attacks last
erythematous nodules, smooth and shiny, a day or two at a time and recur every 36 to
diffuse infiltrate with ill define border, no 48 h. Which of the following is the most
fluctuation or erosion. There were pain on his appropriate time to take blood specimen to
elbow with nerve enlargement. What establish the diagnosis?  malaria
laboratory test would you do to confirm the a. Anytime
clinical diagnosis?  leprae b. At 10.00 – 12.00 AM
a. TPHA and VDRL c. At 10.00 – 12.00 PM
b. Gram preparation d. 30 minutes after fever
c. KOH preparation e. 120 minutes after fever

d. Skin scrapping 12. A very depressed woman took 20 tablets of a


e. Skin slit smear hypnotic drug belonging to her husband
about less than 30 minutes ago. As she was
being taken to the hospital, she developed
hypotension, tachycardia, and tachypneu.
She’s also unconscious. Which of the
following is the most appropriate
management for this patient?
a. Give apomorphine intramuscular 
9. A 40-year-old patient comes with the complaints induce emesis
of uncomfortable stomach & flatulence. From b. Doing gastric lavage
history, the patient was known live in the valley c. Infuse with sodium bicarbonate
of Napu, Sulawesi and he had a fever frequently. d. Give activated charcoal via NGT
On physical examination, his liver was found (nasogastric tube)
enlarged. Which one of the following test can be e. Doing a hemodialysis
used to confirm the diagnosis?  typhoid
a. Blood test
b. Liver biopsy
c. Rectal biopsy
d. Stool examination
e. CT scan

10. A 53-year-old male farmer developed low-


grade fever, abdominal pain, and diarrhea. He
has a skin lesion of black eschar surrounded 13. An outbreak of pulmonary tuberculosis (TB)
by vesicle and edema near his mouth. A week in a dormitory, and this outbreak were
before, his notice many cattle in his farm presumed to be caused by the same source of
infection, based on the results of restriction
fragment length polymorphism (RFLP) examination revealed mild hepatospleenomegaly
analysis and other findings. Which of the and faint erythematous macules.
following health intervention is the most
appropriate for the case above? 16. Which of the following is most likely to have
a. Rehabilitation caused this man’s illness?
b. Prevention a. Vibrio cholerae
c. Curative b. Shigella dysentriae
d. Medication c. Salmonella typhi
e. Environmental Modification
– ebola hemmorhagic fever
d. Entamoeba histolitica
e. Giardia lambia
14. There is an RNA virus that causes deadly
outbreaks of hemorrhagic disease in Africa.
The patents have developed fever, headache,
and muscle pain followed by abdominal pain,
diarrhea, and rash, with both internal and 17. Which of the following specimen is most
external bleeding. In each outbreak, hospital appropriate for case above?
staff became infected. This virus is highly a. Feces
virulent. Which of the following is the most b. Blood
likely transmission for this disease?  ebola c. Urine
a. Contact with blood or body fluid d. Throat swab
b. Transmitted by mosquitoes bite e. Biopsy
c. Transmitted to human from rodent
excreta Question 18-19 are linked to the following case:
d. Aerosol transmission A 44-year old man on a business trip to South
e. Direct contact with cattle product Kalimantan from Jakarta presented to the
emergency department with symptoms of
15. A 31-year-old man presented with complaints diarrhea and abdominal pain. On physical
of high fever, cough, and shortness of breath. examination was found pitting edema of the
Two hours before admission, he experienced abdominal wall and of the legs. A stool specimen
right-sided chest pain when took a deep was sent to the laboratory for culture and for ova
breath or coughed. Chest film showed and parasites. Blood was drawn for a CBC. The
diffused bilateral interstitial pulmonary culture was negative for enteric pathogens. The
infiltrate. Arterial blood gases showed a PO2 patient was found to have eosinophilia. The wet
of 60 mmHg with 91% hemoglobin mounts from the concentrated stool specimens
saturation. The hematologic, serum and liver revealed large, ellipsoidal, operculated parasite
tests were normal. He lives near the poultry, eggs with thin, transparent shells (140 x 85 µm).
and the pathogenic agent is a new virus.
Which of the following is the prevention for 18. Of the following, which parasite can be the
this disease? cause of this disorder :
a. Vaccine a. Schistosoma japonicum
b. Antitoxin b. Fasciolopsis buski
c. Antibiotic c. Trichinella spiralis
d. Antiviral : oseltamivir d. Echinococcus granulosus
e. Antidotes e. Clonorchis sinensis

Question 16-17 are linked to the following case: 19. Refer to the previous case, the mechanism of
A 28-year old man presented with 8-day history edema is due to :
of increasing fever, malaise, headache, and a. Host-allergic response to metabolic
constipation. He did not receive any prior wastes of parasite
vaccinations. His vital signs revealed temperature b. Reaction to the parasite as it migrates
390C, BP 130/90, pulse 72x/min. His physical through the tissues
c. Reaction to embolization of eggs oxidase-positive; that resemble fine grains of
d. Immediate hypersensitivity reaction to sand. The patient was probably infected with
the adult worms in the intestine which of the following microorganism?
e. Eggs damage the wall of intestine a. Mycobcaterium tuberculosis
b. Salmonella typhi
20. A 24-year-old man returned from Papua c. Brucella species
Island. Five days later, he developed d. Pasteurella pestis
repeating intense chills and high fevers. e. Staphylococcus aureus
These severe episodes of fever had been
occurring every other day. In between these 23. A 55-year-old warden found a dead muskrat
episodes, he had low-grade fever, myalgia, on the bank of a stream. He picks up the
nausea, vomiting, and diarrhea. A few hours animal and buried it. Four days later, he
ago, he was admitted to ICU in coma. He developed a 1.5 cm painful ulcer on the index
became progressively somnolent and died a finger of his right arm, a 1 cm ulcer on his
week later. Which of the following organs right forehead, and pain in his right axilla.
does this infectious agent initially proliferate Physical examination also revealed right
after entry in the infected host? axillary lymphadenopathy. Which of the
a. Heart following is working diagnosis for this
b. Liver patient?
c. Brain a. Brucellosis
d. Spleen b. Pertusis
e. Renal c. Leptospirosis
d. Tularemia
e. Hydatidosis
21. A 61-year-old man was seen in the
Neurology Clinic for evaluation of a Question 24-25 are linked to the following case:
peripheral neuropathy. He was found to have A volunteer man returning from a tsunami area
dermal hyperpigmentation/depigmentation was admitted to hospital. A week before, he had
(salt/pepper) of the skin, scaly palms, and fever, headache and myalgia especially at calf.
transverse ridges on his fingernails which These symptoms resolved, but the day before
were identified as being a result of a chronic admission he became pyrexia and on examination
chemical intoxication. was found to be jaundice and to have an elevated
Which of the following is the most likely blood urea. Urine was collected and inoculated
chemical agent caused that sign and into a semisolid agar medium, and examined by
symptoms? dark ground microscopy.
a. Acetaminophen
b. Arsenic : ngumpul di rambut kuku 24. Which of the following is your working
c. Cyanide diagnosis?
d. Fluoride a. Syphillis
e. Iron b. Pertusis
c. Weil’s disease
22. A 45-year-old man from the Middle East had d. Lyme disease
fever and chills, with weight loss, sweats, e. Relapsing fever
headache, muscle pain, fatigue, and
depression. From physical examination the
doctor found lymphadenopathy and 25. Which of the following disease is the
spleenomegaly. The man is a daily farmer complication of this disease?
and a couple weeks before the symptoms a. Pulmonary hemorrhage
appeared he drank a glass of unpasteurized b. Myopericarditis
cow milk. The culture of blood grew a tiny c. Aseptic meningitis
Gram-negative coccobacilus, catalase and d. Osteomyelitis
e. Anemia subsequent recurrences. They report the
disease to district public health office. The
26. A 39-year-old woman live in Manokwari, investigation found that one of the food
Papua Island developed a major seizure while products eaten by this tourist was
at work. She had no history of epileptic contaminated by suspected pathogens. What
disease. A head MRI was remarkable for a is the suspected pathogen may cause the
lesion surrounding a scolex. The etiologic disease above?
agent would most plausibly have been a. Salmonella typhi
acquired by eating or dinking which of the b. Shigella dysenteriae
following food items? c. Enterohemorragic E. coli
a. Uncooked vegetables d. Staphylococcus aureus
b. Raw beef e. Enterotoxigenic E.coli travellers
c. Raw pork 30. A 23-year-old man sees his family physician
d. Uncooked fish with a sudden onset of 4-day history of
e. Unfiltered water fevers, headache, retro-orbital pain, myalgia
and rash. Physical examination shows diffuse
27. A young boy had wart-like skin lesions with erythroderma with blanching erythema and
the appearance of strawberries in his legs. petechial formation resulting from pressure
Later, this lesion destroys the bone of his legs applied to her skin. Laboratory test reveal
(gummata), but there are no visceral and lymphopenia and thrombocytopenia. Which
nervous system complications. The disease is of the following virus is most likely
still endemic in hot tropical countries, such as responsible for this infection?
Indonesia. Which of the following a. Morbili
examination is the most appropriate to b. Dengue
diagnosis the disease above?  yaws c. Influenza
a. Widal test d. Coxsackie
b. IgM-IgG of Treponema e. Rhinovirus
c. Dark-field microscope
d. FTA-ABS and TP-PA test 31. A 48 year old lady came to the outpatient
e. ELIZA assay clinic with chief complaint of fatigue and
difficulty in concentration for the last 2
28. A 27-year-old man develops acute severe weeks. The history taking revealed that she
encephalitis that requires hospitalization. had a balanced diet, no history of chronic
Several days before he got that disease, he disease and had been a heavy smoker for 20
recognizes that many pigs in his farm were years. On physical exam it was found that she
died. had anaemic conjunctiva with other findings
Which of the following is most appropriated was within normal limit. The laboratory
natural host for the pathogen?  nipah examination showed that she had haemolytic
a. Arthropod anemia.
b. Horse Based on the information given above, what
c. Bats is the most likely cause for her condition?
d. Squirrel a. Vitamin K deficiency
e. Rats b. Copper deficiency
c. Vitamin E deficiency
29. A female tourist developed gastroenteritis d. Iron deficiency
while visiting small town in Indonesia and e. Folate deficiency
tried some Indonesian traditional food. The
onset of the disease is abrupt with abdominal 32. A laboratory examination of a leprosy patient
cramps and watery diarrhea. She had no after 4 months MDT therapy revealed
fever or nausea or vomiting. The symptoms Bacterial index 6+ and morphological index
have resolved within 24 hour and no 50%. What is your conclution for this result ?
a. Pausibacillar leprosy patient sensitive to a. Reye syndrome
treatment b. Gray syndrome
b. Multibacillar leprosy patient sensitive to c. Down syndrome
treatment d. ADHD
c. Multibacillar leprosy patient resistent to e. Black water fever
treatment
d. Pausibacillar leprosy patient resistent to 36. A 35 year old man, a businessman, came to
treatment private doctor. He asked the physician about
e. Pausibacillar leprosy patient prone to his plan to go to Papua, which is endemic
nerve damage malaria. The physician suggested him to take
prophylactic drug. Which medication (WHO
33. A boy brought by his mother to the recommended) would be given to this patient
emergency department of Siloam hospital for prophylactic?
because of vomit & dyspnoea. According to a. Kina
his mother, her son was playing in the b. Chloroquine
backyard. On physical examination, the boy c. Doxycycline
was found to have high fever (39 ºC) and d. Kuinin
multiple stinging on the back. The most e. Primakuin
likely species of insect bites is:
a. Hymenoptera sp 37. A 40 year old man came to hospital with four
b. Dermacentos andersoni: tick paralysis times generalized tonic-clonic seizure since
c. Lactodectus mactan : black widow two week ago. There was no history of
d. Loxoxceles laeta : laba2 seizure and head trauma. A one year ago, he
e. Lytta vesicatoria : kumbang had business in Papua for six months. During
his stay in there, he often consumed under-
cooked pork. Brain MRI showed a small
34. A 28 year old lady came to the Posyandu to cystic with invaginated scolex inside. What
have a nutrition consultation. Last year, she of this following is the most likely inside
delivered a stillbirth baby with a spinal cyst?
disorder. She and her husband start planning a. Taenia eggs
to have a baby again. She asked for b. Cycticercosis cellulose
suggestion about her dietary intake before she c. Cysticercosis bovis
got pregnant. d. Taenia solium worm
What kind of diet does she need to prevent e. Taenia saginata worm
the same disorder for her future baby?
a. Dark green vegetables
b. Pasteurized milk 38. An 8-day-old infant was born with the help of
c. Deep water fishes shaman (‘dukun beranak’). The mother states
d. Boiled eggs his son became irritable since 2 days ago, and
e. Lean beef now any loud noise appears to cause him
pain, as evidenced by muscle tightening and
35. A 27 year old woman in at term pregnancy back arching causing his head to nearly touch
had fluctuated fever since 2 weeks ago. She his feet. Physical examination reveals only a
looked pale, generalized weakness, and dried packing on his umbilical cord, as is the
headache. She went to public health center local custom. He appears normal until he is
and was prescribed oral chloramphenicol in stimulated by touch or a loud noise, and then
doses of 500 mg four times daily and he begins to cry, stiffens, and arches his back.
paracetamol 500 mg three times daily. She The stiffness continues until he calms down.
did not tell to the physician that she was Of the following, the MOST likely diagnosis
pregnant. What is the expected side effect of is:
the drug you will find on the baby? a. Bacterial meningitis
b. Botulism duration. On examination she is very ill-
c. Generalized seizure appearing. She has some inspiratory stridor
d. Tetanus and thick white-gray material covering her
e. Viral encephalitis tonsils and faucial pillars, and she has
swelling of her neck, no splenomegaly. The
Question 39-40 are linked to the following case: lymphocyte is normal. What is the likely
A worried mother brings her 18-month-old son to diagnosis?
the emergency department because of a rash that a. Tonsilitis
developed today. She reports that he has had a b. Angina Plaut Vincent
runny nose, conjunctivitis, and diarrhea. On c. Mononucleosis infectiosa
physical examination, he appears severely ill, is d. Laryngitis
temperature to 40°C for the last 3 days and has e. Diphteria
diffuse 2 – 3 mm erythematous rash beginning
from the hairline behind the ear and spreading to 43. A baby is born with a rash identical. History
his face and body. On buccal mucosa, there is a taking reveals that the mother had a febrile
red spot surrounded by white bluish appearance. illness during the second trimester of
pregnancy. Examination reveals diffuse
39. Of the following, the MOST likely cause of raised purple skin lesions. There is no pallor,
the rash is jaundice, or cyanosis. The baby has cataract,
a. Rubella virus a 3/6 systolic heart murmur, and enlargement
b. Coxsackievirus of both liver and spleen. There is no
c. Morbilli virus lymphadenopathy.
d. Human herpesvirus 6 Of the following, the MOST likely diagnosis
e. Parvovirus B19 is?
a. Congenital Toxoplasmosis
40. What is the most severe complication that b. Congenital Rubella
would happen to the patient above? c. Congenital Cytomegalovirus
a. Osteomylitis d. Congenital Herpes Simplex
b. Encephalitis e. Congenital Varicella
c. Otitis externa
d. Sinusitis 44. A 40-year-old woman presented with a raised
e. Oral thrush redness plaque resemble a doughnut with a
raised border that she already had for 1 year,
41. A 48-year-old man with acute gastroenteritis It was no itchy, no pain, not easily bleeding.
has step ladder night fever since 7 days ago. Sometime she feel weakness on her leg and
He also has abdominal pain, and constipation. her slipper left behind while walking. On her
He usually eats uncooked vegetables at street legs she developed an anular erythematous
vendor (pedagang kaki lima). sharp border lesion, no scally, shiny surface
Which of the following is most likely to be a and enlargement of the popliteal nerve. What
constituent of this organism? kind of laboratory examination results that
a. Vi antigen can help us make a diagnose of the case
b. Urease above?  kusta
c. Hemolysin a. Dark field examination
d. Shiga toxin b. Gram
e. Pili c. KOH examination
d. Ziehl neelson
e. Na Cl examination

45. A woman, recently returned from Lampung,


42. A 10-year-old girl presents with a history of complains of having paroxysmal attacks of
sore throat and difficulty breathing of 1 day’s chills, fever, and sweating; these attacks last
a day or two at a time and recur every 36 to in all other extremities and a normal cranial
48 h. Examination of a stained blood nerve examination. Which of the following
specimen reveals ringlike and crescent-like vaccination is the best prevention for this
forms within red blood cells. The infecting disease?
organism most likely is a. MMR
a. Plasmodium falciparum b. HBIG
b. Plasmodium vivax c. BCG
c. Plasmodium ovale d. DPT
d. Plasmodium malarie e. Oral Sabin  polio
e. Plasmodium brasilianumi
49. An HIV (+) patient comes to the emergency
room with severe dehydration. The nurse puts
an IV line on him, after that the nurse use her
two hand recapping the needle, and got
46. A 33-year-old woman ingested 10 tablets of injured by that needle. The nurse is
an analgesic-antipyretic which is metabolized panicking. You are the doctor on duty in that
to a quinone imine. Shortly after ingestion emergency room. What should you do?
she presented with signs of nausea, vomiting, a. Press the injury site to stop the blood
and generalized malaise. Within 3-5 days b. Wash immediately using soap under
peak hepatic dysfunction occurred and was running tap water
associated with hepatic necrosis. c. Use bleach or iodine to clean the injury
Which of the following is the most likely site
specific antidote that should be used to treat d. Use antibiotic ointment on the injury site
this patient's over dosage? e. Enlarge the injury site by incision to let
a. Amyl nitrite the blood flow
b. Atropine
c. BAL
d. EDTA
e. N acetyl sistein

47. A 27-year-old medical student was admitted 50. A 33-year-old woman at 10 weeks pregnancy
to hospital because of sudden onset fever up presents for her first prenatal examination.
to 390C and headache. Two weeks previously Routine labs are drawn and her HBsAg is
he volunteers cleaning the canal with others. positive. Liver function tests are normal and
On examination, he looked so yellow. Blood her Anti HBc and Anti HBs are negative.
tests done shortly after admission indicated Which of the following is the best way to
renal function abnormality and elevated liver prevent neonatal infection?
function tests. Which of the following is the a. Provide immune globulin to the mother.
prevention after exposure for this case? b. Provide hepatitis B vaccine to the
a. Human vaccine mother.
b. Health education c. Perform a cesarean delivery at term.
c. Vector control d. Provide hepatitis B vaccine to the
d. Isolation the infected people neonate.
e. Prophylaxis antibiotics e. Provide HBIG and the HepB vaccine to
the neonate
48. A 24 year old woman admitted to the hospital
with myalgia and weakness of her right leg. 51. A 34-year-old G2 at 36 weeks delivers a
She had traveled from Saudi Arabia and her growth-restricted infant with cataracts, patent
immunization had never completed. On ductus arteriosus, and sensorineural deafness.
examination, she had normal sensation of the She had flu-like syndrome with rash in early
right leg, with normal movement and strength
pregnancy. What is the most likely causative d. Delayed reactions
agent? e. Quantal reactions : dosis berapapun
a. Parvovirus langsung kena : chloramphenicol
b. Rubella virus 55. Male, 46 years old came to you as a doctor
c. Morbili virus with chief complains shortness of breath
d. Cytomegalovirus especially at the work place. When he gets
e. Herpes simplex virus home, the symptom is missing. He also
complains that he sneezes every morning.
52. A 27-year-old female has just returned from a According to him, his sister is using
trip to Southeast Asia. In the past 24 hours, bronchodilator inhaler every day. He just
she has developed shaking, chills, and a worked at sandblasting company and expose
temperature 40o C. A blood smear reveals to dust since 2 weeks ago. Which of the
Plasmodium vivax. Which of the following following is the most appropriate to measure
agents should be used to eradicate the extra- the toxicity of hazardous material in the case
erythrocytic phase of the organism? above?
a. Primaquine a. Body Mass Index
b. Pyrimethamine b. Nutrition and Job description
c. Quinacrine c. Individual Susceptibility
d. Chloroquine d. Social pressure
e. Chologuanide e. Tissue and organs

53. A 25-year-old G3 at 39 weeks delivers a 56. The patient was a 3-year-old girl who was
small-for-gestational-age infant with seen by her pediatrician for a routine physical
chorioretinitis, intracranial calcifications, examination. Her mother was concerned
microcephaly, jaundice, and about her daughter’s poor appetite. Physical
hepatosplenomegaly. The infant displays exammmination revealed that the child was
poor feeding and tone in the nursery. The small for her age and had a slightly enlarged
patient denies eating any raw or undercooked liver. Blood was collected for a routine
meat and does not have any cats living at complete blood count, since she had
home with her. She works as a nurse in the previously been slightly anemic. Her
pediatric intensive care unit at the local hemoglobin level was within the normal
hospital. What is the most likely causative range; however, she did have eosinophilia
agent? (20%). The child had no history of travel.
a. Cytomegalovirus When questioned about pets, her mother
b. Epstein-Barr virus reported that she spent a great deal of time
c. Hepatitis B virus with her puppy. Of the following, which
d. Parvovirus manifestation of the infection (above case) is
e. Yellow fever virus known to occur?  toxocara
a. Creeping eruption  strongyloides
54. A 45 year old man presented chronic skin b. Biliary cirrhosis
lesions on both hands for the past 1 year. On c. Cholecystitis
examination, his skin appears erythematous, d. Pruritus ani  oxyuris
crusted, fell itching, and having serous vermicularis/enterobius
discharge. He was working in the gold e. Visceral larva migrans
refining plant for the past 2 years, works with
potassium cyanide and never uses any gloves. 57. A patient complains of having nail size
Which of the following mechanism is excrement from his anus. On the stool
responsible for this illness? examination, parasite eggs contains a hexacanth
a. Immediate respond embryo with six hooklets surrounded by radially
b. Acute respond striated spherical shell, 30 to 40 u in diameter
c. Gradual reactions : dikit2 : sulfonyl urea
were found, the most likely diagnosis of this c. Fecal – cutaneous transmission
patient is : d. Direct contact with skin scales
a. Saginata taeniasis e. Intravenous drug abuse
b. Solium taeniasis
c. Cysticercosis 61. A 40-year-old man came to the clinic with
d. Oxyuriasis complaints of having fever more than one-week,
e. Ascariasis nausea, and fatique. He came back from
Samarida, Kalimantan. On blood smear
examination, there are late trophozoites forms,
58. The mother of a 4-year-old child notes that schizonts with 12-24 merozoites, and the infected
her child is sleeping poorly and scratching his red cell is enlarged. Of the following, the most
anal area. You suspect the child may have likely diagnosis of this pasient is :
pinworms. Which one of the following is the a. Vivax malaria
BEST method to make that diagnosis? b. Falciparum malaria
a. Examine the stool for the presence of c. Malariae malaria
eggs d. Ovale malaria
b. Examine the stool for the presence of e. Knowlesi malaria
larvae
c. Examine a blood smear for the presence
of microfilariae 62. A- 52- year old man is brought to the emergency
d. Examine transparent adhesive tape for department in an unconscious condition. On
the presence of eggs examination, he was death form more than one
e. Determine the titer of IgE antibody hour. On autopsy of the brain, there is cyst‘s fluid
against the parasites containing protoscoleces with hooklets and many
daughter cysts were found. The most likely
diagnosis of the disease is:
Question 59-60 are linked to the following case: a. Hydatidosis
A 30-year-old woman presents with abdominal pain b. Cysticercosis
and diarrhea of 3 day’s duration. She does not c. Cerebral malaria
complain of nausea, vomiting, or fever. She has no d. Occult filariasis
sick contacts or significant travel history. A stool e. Trichinosis
sample is obtained, which reveals rhabditiform(,
filariform: infective) larva strongyloides .
63. An-6-year-old girl sustains a large laceration
contaminated with dirt after falling from her
59. In condition below adult form of the pathogenic bike. Her mother can’t recall how many
agent can be found in whole digestive tract and doses of immunization her daughter has
its larvae can be found in visceral organ (lung, received. Which of the following is the
liver, gall bladder) is: prevention after exposure for this case?
a. Eosinofilia a. Adult-type TT
b. Hyperinfection b. TIG
c. Retrofection c. TT and TIG
d. Hypereosinofilia d. HiB containing TT and TIG
e. Autoinfection e. DPT

60. Refer to the previous case, further questioning Question 64-65 are linked to the following case:
reveals that the woman frequently gardens in her A 23-year-old man presents with extreme
backyard. Of the following, which one is the swelling of his legs and scrotum. The skin
transmitted form? associated with the swollen areas is thick and
a. Fecal – anal transmission scaly. The patient admits to an episode of fever
b. Fecal – oral transmission associated with enlarged inguinal lymph nodes
some time ago, but did not think much of it.  e. Rate of decline in anti-HIV antibody
filariasis 68. A patient come with an itchy recured fungal
infection on the feet and need some advice to
64. Which one of the following is the BEST be cured and some prevention measures so
method to make that diagnosis? the diasease will not occure again, doctor
a. Examine the stool for the presence of already gave him some medication for fungal
eggs infection. What kind of advise can make him
b. Examine the stool for the presence of healthy?
larvae a. Don’t eat seafood
c. Examine a blood smear for the presence b. Use always sandals
of adult worm c. Keep the skin dry and clean
d. Determine the titer of IgE antibody d. Don’t use shoes all day long
against the parasites e. Don’t scratch, will spread the infection
e. Examine a blood smear for the presence
of microfilaria 69. A woman came with the complaints of being
uncomfortable and hardened in the left upper
side abdomen. Every four days, she is
65. According to the case above, which of the intermittently feverish. On blood smear
following is the most appropriate time to take examination Plasmodium parasite +. Fever
the specimen for established the diagnosis? symptom on the previous question referred to
a. Anytime :
b. At 10.00 – 12.00 AM a. Saorozoites is in the the blood
c. At 10.00 – 12.00 PM b. Hypnozoites in the liver is formed
d. 30 minutes after fever c. Merozoites discharged from schizonts
e. 120 minutes after fever enter the blood
d. The process of gametocytes in the blood
66. At night, a 28-year-old man is brought to the e. Pigment in the parasite is formed
emergency department after his right foot
being bitten by an animal, within 1 hour 70. A 35-year-old man as a farm worker who was
before admission. The patient said that he working with pesticides is brought to the
heard the sound of the animal. He complains emergency room with headache, vomiting,
of pain in his right foot and there are two salivation, diarrhea, muscle fasciculation,
bitten marks, surrounded by edema and difficulty walking, and difficulty speaking.
erythema in the dorsal aspect of his right His clothing has been removed, he has been
foot. Initial management of the case washed, and he has been given activated
described should be charcoal. What is the most effective
a. Observation remaining treatment for this case of pesticide
b. Cryotherapy poisoning?
c. Antivenin : antivenom a. Epinephrine
d. Incision and suction b. Antacid
e. Corticosteroids c. Spironolactone
d. Atropine
67. A 29-year-old man has weight loss, white e. Hidrochlorothiazide
plagues on the pharynx, and purple lesions on
the abdomen, which on biopsy reveal Kaposi 71. A 65-year-old-woman suddenly had flaccid
sarcoma. Which of the following is the most paralysis of her left leg. A few days ago, she
predictive of the patient’s prognosis?  HIV had headache, fever, sore throat, and nausea.
a. CD4 cell count She lives with her son; daughter in law and a
b. CD4/CD8 cell ratio young grandchild who’s just received a
c. Degree of lympadenopathy routine oral vaccination. She is taking
d. Level of HIV-1 RNA in plasma immunosuppressant for her kidney’s
transplant. Her vital signs and cranial nerve Based on the information given above, what
examination are normal. A head CT scan and is the most likely cause of her complaint?
lumbar MRI are also normal. a. Protein deficiency due to prolonged
What is the most likely transmission of her breastfeeding
infection? b. Carbohydrate deficiency due to increased
a. Droplet calorie need
b. Air borne c. Fat soluble vitamin deficiency due to
c. Fecal-oral high vegetable/fruit intake
d. Close contact d. Water soluble vitamin deficiency due to
e. Animal bite cooking process
e. Mineral deficiency due to competition
with high vitamin absorption
72. A pork-eating village in the highlands of
Papua New Guinea is reported to be suffering 75. A 10-year-old boy who has moved to your
from an epidemic outbreak of epileptiform practice recently has sore throats, get high
seizures. You have been sent to investigate. temperature. The child reports a runny nose,
One of the first things you should investigate mild cough, and abdominal pain. Findings on
is? physical examination include a temperature
a. The level of Balantidium coli in swine of 38°C, and vesicular lesions on the soft
stool palate, dorsal and palmar of hands. There is
b. The practice of consuming raw deceased no cervical adenopathy or rash.
human brains Of the following, the MOST likely diagnosis
c. The presence of taenia eggs in the is  hfmd
drinking water a. Adenovirus infection
d. The presence of tropozoites in the human b. Coxsackie virus infection
blood c. Mononucleosis
e. The quantity of culidae in the village d. Sinusitis
e. Streptococcal pharyngitis
73. A 34-year-old woman complained a sudden
onset of high fever for 4 days with nausea,
vomiting, headache, muscle ache. There were 76. A 19-year-old woman comes to Emergency
petechiae on examination. Which of the Department with signs and symptoms of
following laboratory result is most poisoning of an agent include nausea,
appropriate with the patient’s diagnosis?  vomiting, abdominal cramps, diarrhea,
dhf excessive salivation, headache, giddiness,
a. Trombositopenia rhinorrhea, tightness in chest, pin-point
b. Neutropenia pupils, mental confusion, and muscle
c. Anemia twitching. Which of the following is the most
d. Decreased of hematocrit : arusnya naik likely agent caused those symptoms?
e. Leucopenia a. Acetaminophen
b. Barbiturate
74. A 1 year old girl came to the Puskesmas with c. Carbamate
swollen gum for a week. From the history d. Cyanide
taking it was found that she was born normal e. Opiate
with normal birth weight, had been breastfed
up to now and had a balanced diet with high 77. A 23-year-old G1 with a history of a flulike
intake of cooked vegetable and fruit since 6 illness, fever, myalgias, and
months old. She had no history of chronic lymphadenopathy during her early third
diseases. Her other physical and laboratory trimester delivers a growth-restricted infant
examination was normal. with seizures, intracranial calcifications,
hepatosplenomegaly, jaundice, and anemia.
She has many pets in her house. What is the e. Gall bladder
most likely causative agent? torch
a. Trepanema pallidum TROPICAL MEDICINE – MCQ 2009
b. Trichomonas vaginslis
c. Toxocara canis  ocular dan visceral 1. A young man was admitted to hospital after
larvae migrans increasing left arm pain and paresthesia.
d. Neissheria meningitidis Several days ago, he contacted with a dog.
e. Toxoplasma gondii His symptoms increased and were
accompanied by hand spasm and sweating on
78. A 22-year-old man who works in the the right side of the face and trunk. This
pediatric ward of a hospital suffers from patient was admitted to the hospital the day
malaise, sneezing, and runny nose. He after developing dysphagia, hypersalivation,
subsequently develops a mild sore throat, agitation, and generalized muscle twitching.
headache, and stuffy nose. The symptoms Which of the following could rapidly destroy
resolve within 4 days. Which virus is most the pathogenic agent of this disease?
likely to be responsible for these symptoms? a. Infrared radiation
a. Rota virus b. Catalase
b. Rubella virus c. Alkali water
c. Coxsackie virus d. Sunlight
d. Hepadnavirus e. Heating at 60ºC for 30 minute
e. Influenza virus
Question 2-3 are linked to the following case:
79. Approximately 4 hour after eating a meal in A 65-year-old-woman suddenly had paralysis of
restaurant, 3 members of a tourisms group her left leg. A few days ago, she had headache,
develop a sudden onset of nausea, vomiting, fever, sore throat, and nausea. She lives with her
severe abdominal cramps, and diarrhea. son; daughter in law and a young grandchild
Nobody got febrile. Which of the following who’s just received a routine immunization. She
vaccination is should be given to the traveler? is taking immunosuppressant for her kidney’s
a. Shigella vaccine transplant. Her vital signs and cranial nerve
b. Salmonella vaccine examination are normal. A head CT scan and
c. Enterobacter vaccine lumbar MRI are also normal.
d. H. pylori vaccine
e. ETEC vaccine 2. What is the most likely transmission of her
infection?
a. Droplet
b. Air borne
80. An outbreak investigation note many c. Fecal-oral
costumer from a café were admitted to the d. Close contact
hospital. They have fever; nausea-vomiting, e. Animal bite
constipated or diarrhea, weakness and altered
mental status. Rose spots are seen on the 3. Which of the following is the primary
trunk. Blood cultures from patients grow a pathologic effect in this case?
non-lactose-fermenting gram-negative rod. a. Persistent viremia
These people are infected from the carrier b. Focal multiplication
person c. Immune complexe formation
In which of the following sites are bacteria d. Destruction of infected cells
most likely to be found in carrier person? e. Reduce of acethylcholine receptor
a. Blood
b. Kidney Question 4-5 are linked to the following case:
c. Liver
d. Intestine
A young boy is suffering from high fever, 8. A 2 year old child presented with a history of
difficult to swallow water, hydrophobia, and febrile upper respiratory tract infection. His
disorientation. On examination, there is a clinical conditions deteriorate with a
hypoesthesia on his left arm. Four weeks ago, he worsening cough and dyspneu. He was
had been bitten by a dog. The characteristics of admitted to the hospital. On clinical
etiologic agent were zoonotic, single-stranded, examination, the child had obvious evidence
negative-sense DNA virus. of respiratory distress with rib retraction and
an elevated respiratory rate. A chest x-ray
4. Which of the following is usefull for making showed hyperinflation of both lung fields.
a diagnosis in this case ? What is the most appropriate specimen for
a. Giant cell laboratory diagnosis in this case?
b. Limfosit plasma biru a. Nasopharyngeal washing
c. Negri bodies b. Tracheal aspirate
d. Inclusion bodies c. Bronchial washing
e. Clue cell d. Nasal swab
e. Blood
5. Which of the following is available for
treating this patient? Question 9-10 are linked to the following case:
a. Immune globulin A 31 year old female underwent allogenic bone
b. Live attenuated vaccine marrow transplantation for acute myeloid
c. Antiviral leukemia. In the second month post transplant,
d. Antimicrobial she developed high fever, a nonproductive cough
e. Antitoxin and breathlessness. Physical exam showed
splenomegaly, and laboratory studies showed
Question 6-7 are linked to the following case: anemia, leucopenia, and thrombocytopenia, and
A 24 year old woman admitted to the hospital abnormal liver function test. Her chest x-ray
with myalgia and weakness of her right leg. She showed diffuse interstitial pulmonary infiltrates.
had traveled from Saudi Arabia and her A bronchoalveolar sample performed infected
immunization had never completed. On kidney cells showed owl eye.
examination, she had normal sensation of the
right leg, with normal movement and strength in 9. What is the best treatment for this infection
all other extremities and a normal cranial nerve disease?
examination. a. Antimicrobial
b. Antiviral
6. Which is a target cell receptor for this c. Antitoxin
pathogenic agent? d. Immunosuppressant
a. Silica acid on epithelial cells e. Antifungal
b. Acetylcholine on neurons
c. CD4 molecule on T lymphocytes 10. This female’s illness could be most easily
d. C3d complement on B lymphocytes diagnosed by which of the following tests?
e. Immunoglobulin on epithelial cells a. HSV-1 IgM antibody
b. CMV antigenemia
7. Which of the following is the best specimen c. HHV-6 IgM antibody
for diagnosis in case above? d. Culture of blood in erythroblastic cells
f. LCS e. Monospot test
g. Blood
h. Urine 11. In an elementary school many of students
i. Throat swab suffer from sore throat, runny nose, cough,
j. Rectal swab fever, headache, myalgia, and fatigue. The
students who have not been vaccinated
against this infection are offered prophylactic
treatment with certain antiviral drug. What is a. Sickle cell trait decreased the patient’s
the mechanism of action for this drug? resistant to the virus infection
a. Blockade of nucleocapsid release b. Sickle cell trait promotes infection of
b. Blockade of viral attachment erythrocytes by plasmodium
c. Blockade of viral protein synthesis c. Virus infection promotes INF production
d. Inhibition of RNA-dependent DNA d. Virus infection targets erythrocyte
polymerase precursor cells
e. Inhibition of RNA-dependent RNA e. Bacterial infection activates leukocytes
polymerase cells

12. A 28 year old female nurse became ill after 15. A 26-year-old nulliparous woman, in her
taking care of a patient with pneumonia 2 seventh month of pregnancy complains of a 7
days ago and was admitted to the hospital days history of fever especially at night. As
with fever and sore throat. She was placed in she recall, 2 weeks ago she ate gado-gado in
respiratory droplet isolation and quickly coastal area when she out for duty for several
developed respiratory failure, requiring days. She also had diarrhea and abdominal
mechanical ventilation. In 10 days, nine other discomfort. On physical examination reveals
patients in the same ward developed the same blood pressure 110/70 mmHg, heart rate 60
symptoms. Which of the following laboratory ×/min, RR 20x/min, temp. 39.1°C. Tongue is
test is most appropriate? coated and tremor. Fetal USG shows normal.
a. Blood culture to identify S.pneumoniae What is the most likely etiologic factor
b. Urinary antigen testing for Legionella above?
pneumophila a. Salmonella typhi
c. Virus culture to identify influenza A b. Plasmodium falciparum
d. Serology to demonstrate antibodies to c. Vibrio cholera
identify SARS d. Hepatitis A
e. PCR of nasopharyngeal secretion to e. Escherichia coli
identify RSV

13. A young man presented with headache, low 16. A 4-year-old child presents with fever,
grade fever, and painful blisters on the penis. cough, conjunctivitis, coryza, photophobia,
He admitted to having had unprotected sex in posterior cervical adenopathy. Red lesions
the past month. Physical examination with a white center are present on the buccal
revealed inguinal lypmadenophaty and mucosa. A generalized erythematous rash is
blisters on the penis. Which of the following also noted. What is the most likely diagnosis?
pathogens is most likely to have caused his a. Rubella
disease? b. Kawasaki disease
a. Human papilloma virus c. Adenovirus infection
b. Haemophillus ducreyi d. Rubeola
c. Treponema pallidum e. Varicella
d. Chlamydia trachomatis
e. Herpes simplex virus 17. A 35-year-old man as a farm worker who was
working with pesticides is brought to the
14. A 4 years old boy with sickle cell disease is emergency room with headache, vomiting,
taken to his pediatrician’s because he is pale salivation, diarrhea, muscle fasciculation,
and is suffering from repeating intense chills difficulty walking, and difficulty speaking.
and daily high fever. A blood test shows that His clothing has been removed, he has been
WBC was 17.000/µl and a platelet count is washed, and he has been given activated
53.000/µl. What description is relevant to the charcoal. What is the most effective
findings in this patient? remaining treatment for this case of pesticide
poisoning?
a. Epinephrine c. Absence seizure
b. Antacid d. Atonic seizure
c. Spironolactone e. Secondarily generalized seizure
d. Atropine
e. Hidrochlorothiazide 21. What is the most appropriate antiepileptic
drug for case above?
18. Mr. Farid is a diabetic. He read in a book that a. Carbamazepine
the main source of the brain fuel is glucose. b. Valproic acid
His doctor said that insulin in his body is not c. Benzodiazepine
enough for entering glucose to the cell. So he d. Lamotrigine
worries about the work of his brain. You e. Ethosuximide
know that the brain is protected from high
molecule particle by blood-brain barrier Question 22-23 are linked to the following case:
(BBB). What do you think?
a. The main brain fuel is not glucose A 70 years old man presents to ER with history
b. Glucose is not big enough to be block by of seizure. His wife awakened at 05.00 by an odd
blood-brain barrier gurgling noise. Her husband’s head was deviated
c. Glucose enter the brain through the to the left and his left arm was stiffened. After a
leakage part of the BBB few moments he had generalized body jerking
d. There is special transporter for glucose to and was unresponsive. Event lasted 2 minutes but
enter the brain stopped spontaneously. His wife said he seemed
e. Glucose in the only exception for blood drowsy and confused. There was no history of
brain barrier prior seizure. On ER, he was conscious, BP was
140/95 mmHg, HR 96 x/min, RR 20 x/min,
19. A 45 year old man has chronic otitis media Temperature 36, 5 C. The laboratory examination
since 2 years ago. He comes to Siloam revealed Random Blood Glucose was 610 mg/dl.
hospital because he suffers headache since a
week ago. The doctor makes some 22. What is the most likely diagnosis?
examinations also radiographic examination a. Pseudoseizure
on his head. The doctor fined a round mass b. Primary epilepsy
with 3 cm diameter. Which of the most likely c. Secondary epilepsy
cause of the mass? d. Acute provoked seizure
a. Neoplasma e. Generalized seizure
b. Aneurisma
c. Infarction 23. What is your treatment for case above?
d. Abcess a. Diazepam intravenous
e. Hemorrhage b. Oral phenytoin
c. Ringer lactate infusion
Question 20-21 are linked to the following case: d. Insulin subcutan
e. Dextrose bolus intravenous
A 5 years old boy came to the clinic with
recurrent blanking out at school for 1 month. He
had recurrent episodes in which he abruptly stops
all activity for about 10 seconds, followed by a
rapid return to full consciousness. After the 24. A 5-year-old girl brought by her mother to a
episode patient resumes whatever activity he was clinic because of a-two-day fever and sore
previously engaged with no awareness that throat. On mouth examination, the doctor
anything has occurred. finds a leathery grayish white membrane
20. What is the most likely diagnosis? adheres to pharyngeal walls and tonsillar
a. Simple partial seizure pillars which spread asymmetrically onto the
b. Complex partial seizure
soft palate and uvula. What treatment should
be given to the patient? 27. One member of the NGO (Non-
A. Antibiotic and antiviral Governmental Organization) team from
B. Antiviral and antifungi Indonesia has been reported dead in Mexico
C. Antitoxin and antifungi because of a spider’s bite. The possible cause
D. Antibiotic and antitoxin of this disorder is:
E. antiviral and antitoxin a. Erucism
b. Lepidopterism
25. A 22-year-old woman who just got home c. Tick paralysis
from a 2-week voluntary work in rural Papua d. Arachnidism
comes to a clinic with history of bloody and e. Delusional parasitosis
slimy diarrhea since her last five days in
Papua. She experiences more than five
bowel movements within each day. She also 28. A boy brought by his mother to the
experiences stomach cramp and painful emergency department of Siloam hospital
passage of stool. She reports no fever or any because of vomit & dyspnoea. According to
other complains. There is a slight lower his mother, her son was playing in the
quadrant abdominal tenderness on her backyard. On physical examination, the boy
physical examination. She is found to be a was found to have high fever (39 ºC) and
little bit dehydrated. What is the most likely multiple stinging on the back. The most
organism which causes her condition? likely species of insect bites is:
a. Entamoeba histolityca f. Hymenoptera sp
b. Vibrio cholerae g. Dermacentos andersoni
c. Rotavirus h. Lactodectus mactan
d. Salmonella typhi i. Loxoxceles laeta
e. Escherichia coli j. Lytta vesicatoria

26. A 7-year-old boy presents in emergency unit Question 29-30 are linked to the following case:
with colicky abdominal pain since two days
ago. He also has fever and appears lethargic. 29. A 10 years child came with the complaints of
He has experienced constipation for four days diarrhea, occasionally with blood & mucus. 2
and several greenish vomiting. His vital signs days ago, the patient complained about the
are deteriorating. On physical examination, swelling in the anus. On physical
the boy has abdominal tenderness and the examination was found prolapsed of the
doctor finds palpable abdominal mass in the rectum. On stool examination was found
hypogastric area. Prior to this, he has parasite eggs. The most likely cause is the
recurring abdominal pain and bloody stool worm of species:
for several weeks, has history of passage of a. Ascaris lumbricoides
worm through his rectum, and has been b. Necator americanus
treated with antihelminth.The boy and his c. Angilostoma duodenale
family live in a suburb area and he loves to d. Trichuris trichiura
play marble with his friends. His nails are e. Enterobius vermicularis
dirty and his mother told the doctor that he
often forget to wash his hands before eating. 30. Refer to the case above, the habitat of this
What is urgent further study the doctor worm is located on
should take to make the diagnosis? a. Liver
a. Stool examination b. Lung
b. Complete blood count c. Caecum
c. Blood glucose level test d. Small bowel
d. Plain abdominal radiography e. Stomach
e. Serologic test
Question 31-32 are linked to the following case: a. Examine the stool for the presence of
eggs
A 35-year-old man was brought to the emergency b. Examine the stool for the presence of
department of hospital with seizure. On cerebro larvae
spinal fluid (CSF) examination shows c. Examine a blood smear for the presence
neutrophils, red blood cells, and amoebae form of microfilaria
on wet microscopy. d. Examine transparent adhesive tape for
the presence of eggs
31. Which of the following, which parasite can e. Determine the titer of IgE antibody
be the cause of this disorder : against the parasites
a. Naegleria fowleri
b. Entamoeba histolytica 36. A 23-year-old man presents with extreme
c. Acanthamoeba castellani swelling of his legs and scrotum. The skin
d. Entamoeba coli associated with the swollen areas is thick and
e. Plasmodium falciparum scaly. The patient admits to an episode of
fever associated with enlarged inguinal
32. What is the transmission of this disease? lymph nodes some time ago, but did not think
a. Ingestion of infected snails much of it. Which one of the following is the
b. Food & water contaminated with parasite BEST method to make that diagnosis?
c. Mosquito bites a. Examine the stool for the presence of
d. Sexual contact eggs
e. Intranasal infection while swimming in b. Examine the stool for the presence of
the warm fresh water larvae
33. A woman came with the complaints of being c. Examine a blood smear for the presence
uncomfortable and hardened in the left upper of microfilaria
side abdomen. Every four days, she is d. Examine a blood smear for the presence
intermittently feverish. On blood smear of adult worm
examination Plasmodium parasite +. Of the e. Determine the titer of IgE antibody
following, which parasite can be the cause of against the parasites
this disorder :
a. Plasmodium falciparum 37. A patient came to the clinic with portal
b. Plasmodium vivax hypertension syndromes. From the interview, the
c. Plasmodium malariae patient was known to consume raw fish regularly.
d. Plasmodium ovale Of the following, the most likely cause of disease
e. Plasmodium cynomolgi is :
a. Paragonimiasis
34. An employee who works in Bogor has a task in b. Clonorchiasis
Samarinda for 3 weeks. What is the most c. Fasciolopsiasis
possible drug (s) should be given for this d. Echinococcosis
condition? e. Fascioliasis
a. Chloroquine+ primaquine
b. Sulfadoksin-pirimetamin
c. Atersunate +amodiaquine 38. A 40-year-old patient comes with the complaints
d. Kina+tetracycline of uncomfortable stomach & flatulence. From
e. Doxycycline history, the patient was known live in the valley
of Napu, Sulawesi and he had a fever frequently.
35. The mother of a 4-year-old child notes that On physical examination, his liver was found
her child is sleeping poorly and scratching his enlarged. What is the most likely caused of the
anal area. You suspect the child may have disease?
pinworms. Which one of the following is the a. Schistosoma haematobium
BEST method to make that diagnosis? b. Schistosoma mansoni
c. Schistosoma rodentatum e. Artemisinin
d. Schistosoma japonicum 43. 52- year old man is brought to the emergency
e. Schistosoma mekongi department in an unconscious condition. On
examination, he was death form more than
Question 39-40 are linked to the following case: one hour. On autopsy of the brain, there is
cyst‘s fluid containing protoscoleces with
A 40-year-old man came to the clinic with hooklets and many daughter cysts were
complaints of having fever more than one-week, found. What is the most likely diagnosis of
nausea, and fatigue. He came back from Timika, this disease?
Papua. On blood smear examination, there are late a. Hydatidosis
trophozoites forms, schizonts with 12-24 b. Cysticercosis
merozoites, and the infected red cell is enlarged. c. Cerebral malaria
d. Hstoplasmosis
39. Of the following, the most likely caused of e. Toxoplasmosis
parasite species is :
a. Plasmodium vivax 44. A 27 year old women suffering fever for 2 day.
b. Plasmodium falciparum She brought to emergency department
c. Plasmodium malariae unconsciously and she had general seizure 2
d. Plasmodium ovale hour ago. On physical examination she is
e. Plasmodium cynomolgi comatous, GCS (Glasgow Coma Scale) : 3; BP
90/60 mmHg; t 41 °C. Lab. finding: Hb 10, 3 gr
40. Fever symptom on the previous question %; WBC 5800/mm3; platelet 39.000/mm3;
referred to : there is schizon form on blood smear. Why the
a. Sporozoites is in the blood pathogeniv agent of this disease becoming the
b. Hypnozoites in the liver is formed most dangerous species?
c. Merozoites discharged from schizonts a. It make long term relaps
enter the blood b. Intermittent fever
d. The process of gametocytes in the blood c. Gametocyte formation take place in
e. Pigment in the parasite is formed visceral organ
d. Can make capillary obstruction in Central
Question 41-42 are linked to the following case: Nervous System by parasite contain
erythrocyte
A 30-year-old man, having fever for 3 days, with e. There are many drug resistance against P.
falciparum rings +++ was given CQ3 falciparum
(chloroquine for 3 days). After 3 days, the patient
is still feverish, blood smear examination shows 45. A 35-year old man working on pig farm is
falciparum rings ++++. brought by his employer to the emergency
department of Siloam hospital because of
41. What is the possible response of CQ? seizure and bilateral lower extremity weakness.
a. Sensitivity (S) CT scan of the head reveals several calcified
b. R I resistance, early recrudescence regions. Laboratory examinations show WBC
c. R I resistance, delayed recrudescence count of 10.800/mm3, with eosinophils. Brain
d. R II resistance tissue biopsy reveals scolex with hooklets.
e. R III resistance Which one is the treatment of choice for this
case?
42. Refer to the previous case above, what is the a. Sulfadiazine-pirimetamin
treatment of choice? b. Artemisinin
a. Primakuin c. Albendazol
b. Amodiakuin d. Metronidazol
c. Meflokuin e. Itrakonazol
d. Sulfadoksin pirimetamin
46. A female tourist developed gastroenteritis
while visiting small town in Indonesia and 49. What is the most likely pathogenic agent that
tried some Indonesian traditional food. The causes the patient’s disease?
onset of the disease is abrupt with abdominal a. Bordetella pertussis
cramps and watery diarrhea. She had no b. Haemophilus influenzae
fever or nausea or vomiting. The symptoms c. Coxsackievirus
have resolved within 24 hour and no d. B. Parapertussis
subsequent recurrences. They report the e. C. diphtheriae
disease to district public health office. The
investigation found that one of the food 50. What is the laboratory test to confirm the
products eaten by this tourist was diagnosis in this patient?
contaminated by suspected pathogens. What a. Blood culture
is the suspected pathogen may cause the b. Throat swab culture
disease above? c. Chest X-ray
d. Measurement of IgG antibody serum
f. Salmonella typhi e. Urine culture
g. Shigella dysenteriae
h. Enterohemorragic E. coli
i. Staphylococcus aureus Question 51-52 are linked to the following case:
j. Enterotoxigenic E.coli
A 2-year-old boy was brought to emergency unit
Balaraja Hospital with difficulty in breathing,
choking, gasping, and whoop cough.
Question 47-48 are linked to the following case:
51. What is your most likely diagnosis?
47. An-8-year-old girl sustains a large laceration a. Bordetella pertussis infection
contaminated with dirt after falling from her b. Haemophilus influenzae infection
bike. Her mother can’t recall how many c. Coxsackievirus infection
doses of tetanus toxoid her daughter has d. Adenovirus infection
received. Management of tetanus prophylaxis e. Corynebacterium diphtheriae infection
in this situation of unknown history of prior
doses of tetanus toxoid includes: 52. In this case all the family members should get
a. Adult-type TT the prophylaxis antibiotics as follow:
b. TIG a. Amoxicillin 30-50 mg/kg/day p.o. for 14
c. TT and TIG days
d. HiB containing TT and TIG b. Amoxicillin 30-50 mg/kg/day p.o. for 7
e. DPT days
c. Erithromycin 40-50 mg/kg/day p.o. for
48. If this patient developed seizures, what 14 days
substance has responsible in this patient? d. Erithromycin 40-50 mg/kg/day p.o. for 7
a. Tetanospasmin days
b. Tetanolisin e. Tetrasiklin 25-50 mg/kg/day p.o. for 7
c. Adenylate cyclase toxin days
d. Fillamentous hemagglutinin
e. Pertactin Question 53-54 are linked to the following case:

Question 49-50 are linked to the following case: Male, 46 years old came to you as a doctor with
chief complains shortness of breath especially at
A 3-year-old boy was brought to emergency unit the work place. When he gets home, the symptom
in Tangerang Hospital with stridor, dyspnea, and is missing. He also complains that he sneezes
“croupy” cough. every morning. According to him, his sister is
using bronchodilator inhaler every day. He just complains pain, edema on her left eyelid and
worked at sandblasting company and expose to there is some vesicle in her left forehead. She
dust since 2 weeks ago. complain pain on her forehead 2 days ago,
but vesicle come out 1 day ago.She
53. What are the most possible causes of his sometimes fells headache and fever. Clinical
disease? manifestation are grouped vesicle, eritema
a. Wood base at her left face.What is the diagnose of
b. Quartz this patient?
c. Tobacco smoke a. Varicella
d. Coal b. Herpes zoster
e. Amonia c. Herpes simpleks
d. Dermatitis venenata
54. The doctor gave him sick-leave certificate for e. Dermatitis iritan
2 days and asks the patient to come back
tomorrow to perform spirometry. What is the 58. A 43 years old women, come to
result of spirometry do you expect? dermatovenereology clinic because she
a. FEV1 decreased complains vesicle at genital, and pain, for the
b. FVC decreased first time. Her husband has same problem 2
c. FEV1 and FVC decreased days ago,but it was cured. She is difficult to
d. Normal walk because of pain on her genital and low
e. MVV increased fever. What is the most appropiate treatment
for this patient?
a. Acyclovir 5 x 200 mg, for 7 days
Question 55-56 are linked to the following case: b. Famciclovir 3 x 500 mg,for 7 days
c. Valacyclovir 2 x 250 mg, for 7 days
55. A 2 years old boy brought to d. Acyclovir 2 x 400 mg for 7 days
dermatovenereology clinic because of itching e. Glucocorticoid topical
all of the body. He complains itching on
palm, side of finger, buttock, and abdomen. Question 59-60 are linked to the following case:
Clinical manifestations are papule, vesicle,
pustule, erotion, and excoriation especially at 59. A 34 years old man, come to
finger web, arm, abdomen, side of hand and dermatovenereology clinic, complain itchy
feet, itchy especially at night.His neighbours on his sole. He is a army, that usually
have same problem.He usually play together exercise at dirty land. Clinical manifestations
with his neighbours.What is the diagnose this are burrow 3 cm on his sole, erithema, and
patients? papule. What is the treatment for patient
a. Pediculosis corporis above?
b. Scabies a. Lindane
c. Tinea corporis b. Gamexane
d. Dermatitis atopik c. Albendazole
e. Psoriasis d. Pirantel pamoat
e. Antibiotika
56. What is the best treatment to case above?
a. Sulfur 60. A 30 year old women, come to
b. Gamexane dermatovenereology clinic, because she
c. Crotamiton complains many vesicle on her body, low
d. Lindane fever, muscle pain, and sore throat . She was
e. Permethrine pregnant 3 months. What is the diagnose of
her illness?
57. A 70 years old women, come to a. Varicella
dermatovenereology clinic because she b. Herpes Zoster
c. Herpes simplex given to increase the activity of his
d. Moluscum Contagiosum acetylcholinesterase?
e. Rubella a. Atropine
b. Deferoxamine
61. In epidemiological perspective, what is the c. Dimercaprol
tropical disease? d. Physostigmine
a. Infectious disease in the tropics e. Pralidoxime
b. Any communicable disease
c. Disease which deaths occur in 65. A migrant worker presents to the emergency
children under five room in respiratory distress. He had been
d. Disease which deaths occur in spraying parathion in the fields for several
productive age days, and today he began to feel sweaty and
e. Primarily found in the developing dizzy. By the time he got to the hospital he
world or profoundly impact the was drooling, gasping, and becoming
health of people living in the tropics agitated. These symptoms are due to the
actions of parathion on which of the
62. Conditions that contribute to the risk for following?
becoming infected with tropical diseases a. Acetylcholinesterase
agent: b. Muscarinicreceptors
a. Biological factors related to c. Neuronallipidbilayer
population density, rural vs. urban d. Nicotinicreceptors
living, nutritional status, climate and e. Voltage-gated sodium channels
other environmental factors, as well
as socioeconomic circumstances 66. A 25-year-old woman is despondent that her
b. Genetic factors husband left her for another woman. She
c. Cultural factors attempts suicide by ingesting 25 tablets of
d. Tropical factors extra strength acetaminophen (=650 mg). Her
e. Synergisms on genetic, cultural, and mother finds her and the empty bottle a
immune factors couple of hours later and immediately rushes
her to the emergency room. Which of the
63. Tropical disease category III is group of following drugs will most likely be given to
tropical disease which control strategy this patient?
proven effective, disease burden falling, and a. Acetylcysteine
elimination planned. What are the examples b. Atropine
of disease of this category? c. Penicillamine
a. Leprosy, lymphatic filariasis d. Pralidoxime
b. Malaria, tuberculosis, dengue, e. Protamine
schistosomiasis
c. Ebola, avian influenza, SAR 67. A 46-year-old woman comes to a hospital to
d. Protein calorie malnutrition, pesticide have several bunions removed from her right
intoxication foot. She chooses conscious sedation rather
e. Hepatitis B, pertusis, poliomyelitis than general anesthesia for this procedure.
She is given intravenous midazolam to
supplement the local anesthetics that are
injected into her foot. Midway through the
surgery, she suddenly becomes agitated,
64. A 24-year-old migrant farm worker is rushed combative, and exhibits involuntary
to a nearby emergency room after an movements. The anesthesiologist determines
accidental exposure to parathion. He is in that she is having a paradoxical reaction to
respiratory distress and is bradycardic. the midazolam. Which of the following is the
Which of the following drugs can be
most appropriate drug for counteract that What is the most likely underlying
midazolam? mechanism that affects her nutritional status?
a. Flumazenil a. Increased gluconeogenesis
b. Glucagon b. Increased vitamin excretion
c. Naloxone c. Increased resting energy metabolism
d. Nitrite d. Increased carbohydrate stores
e. Protamine e. Increased protein anabolic

71. A 53-year-old male farmer presents to the


primary health care complaining about a
mass on his arm. He noticed some mildly
itchy red bumps on his arm since a week
before. They started to blister a day or two
68. A 2 year old boy came to the Puskesmas with later and then ruptured. During this time he
ulcer on his left hand. Through the history had a low-grade fever. Futher questioning
taking it was revealed that he had not been reveals he has had no ill contacts and never
well fed in this past year since his mother had anything like this before. He has cows,
died. From his physical examination it was horses, goats, sheep, and chickens on his
found that his BMI (body mass index) was farm. On examination of his right upper arm,
below 5th percentile for boys of his age. He you find a 4, 5 cm circular black eschar
was lethargy and looked pale with a face surrounded by several blisters and edema. He
appearance like a thin old person. His ribs has tender axillaries lymph node
were very prominent. enlargement. A gram stain of fluid drained
Based on this information, he was likely to be from a vesicle and a biopsy from the eschar
classified in which condition below? both show chains of gram-positive bacilli on
a. Marasmus microscopy. What organism is the likely
b. Kwashiorkor cause of this disease?
c. Marasmic-kwashiorkor a. Bacillus anthracis
d. Energy malnutrition b. Bacteriodes fragilis
e. Protein malnutrition c. Borrelia burgdorferi
d. Campylobacter jejuni
69. A one year old boy was diagnosed with e. Yersinia enterocolitica
measles since a week ago. This morning, his
mother noticed that he also suffered for
swollen reddish gums that signified as
scurvy. Question 72-75 are linked to the following case:
What is the most likely mechanism that
underlines his recent condition? At night, a 28-year-old man is brought to the
a. Impaired absorption of micronutrients emergency department after his right foot being
b. Altered micronutrient metabolism bitten by an animal, within 1 hour before
c. Depletion of micronutrient stores admission. The patient said that he heard the
d. Micronutrient redistribution sound of the animal. He complains of pain in his
e. Negative micronutrient balance right foot and there are two bitten marks,
surrounded by edema and erythema in the dorsal
70. An 18 year old girl came to obstetric and aspect of his right foot.
gynecologic outpatient clinic with chief
complaint no menstrual cycle for 2 months. 72. What is the most likely animal that has bitten
She also complained for losing weight 5 kg in the patient?
the last 3 months. From the history taking a. Cottonmouth snake
found that she was treated as a tuberculosis b. Copperhead snake
patient since 4 months ago. c. Rattlesnake
d. Latrodectus mactans C. Stool
e. Centruroides exilicauda D. Lung biopsi
E. Nasofaring swab
73. Initial management of the case described
should be
a. Observation
b. Cryotherapy 4. Pertama2 flu like illness, batuk ga
c. Antivenin sembuh2,vomiting, hoarseness,
d. Incision and suction Answer: pertusis
e. Corticosteroids

74. The next step in the management of the 5. bitten by dog become hydrofobia,
patient should be: hypersalivation, spasm, and sweating.
a. Wide excision
b. Fasciotomy a. giant cell
c. Antivenin
d. Incision and suction b. limfosit plasma
e. Corticosteroids
c. negri bodies
75. After 12 hours, the patient complains of d. clue cell
severe pain especially when he moves his
right leg and his right foot becomes pallor
and cold. There are no pulse in the right
dorsalis pedis and posterior tibial artery. 6. kesalahan banker…kehilangan data.. maap yeii
What is the next management of the case
described should be?
a. Wide excision
A 32 year old worker in a farm having painless
b. Fasciotomy
dermal papule on right hand since 5 days ago.
c. Antivenin
The papule is getting bigger, and the skin become
d. Incision and suction
black. He has tender axilaries lymph node
e. Corticosteroids
enlargement.

Anthrax
SOAL UJIAN TROPIAL MEDICINE
2007
7. Which of the following virulance factor is most
likely to be involve in pathogenesis of illness?
1. Putri…kerjasama nya please…. Demi kalian
juga kok a. exotoxin

2. Putri…kerjasama nya please…. Demi kalian b. endotoxin


juga kok
c. alpha hemolysin
3. Anak dengan panas tinggi, myalgia, dan gejala
flu like syndrome lain, ada susah bernapas. d. lipopolysaccharide
Sodaranya sblm nya juga kaya gt n udah
e. antiphagotic factor
meninggal. Apa spesimen paling baik buat
menegakkan diagnosa?
ɑ. Saliva
B. Blood
8. A 45 years old man was seen in the and recur every 36 – 48 jam. Which the ost
dermatology clinic because of some nodules on appropriate time to take a blood specimen?
his arm&body and sometimes feel pain with
pressure. On examinations, we found a. anytime
erythematous nodules, smooth and shiny, diffuse
inflitrate with ill define border, no fluctuation or b. 10 – 12 a
erosion. There were pain on his elbow with nerve c. 10 – 12 pm
enlargement. What laboratorium test would you
do to confirm the clinical diagnosis? d. 30 min after fever
a. TPHA & VDRL
b. Gram preparation e 120 min after fever
c. KOH preparation
d. Skin scrapping
e. Skin slit smear
12. seorang ibu minum 10 tablet hypnotic 30
menit yang lalu sebelum masuk rumah sakit.
gejala yang tampak : hypotension tachypneu,
9. 40 yo patient complaint uncomfortable tachycardi dan tidak sadar )unconsiusnes.
stomach and flatulence. Live in the valley of tindakan sebagai dokter yang yang pertama kali 
Napu, Sulawesi and he had a fever frequently. dilakukan adalah ..
PE: liver enlarged. Test buat confirm? a. apomorphine
a. Blood b.
b. Liver biopsy c. infusion natrium
c. Renal biopsy d. chancoal dgn NGT
d. Stool exam e.hemodyalisis
e. CT-Scan

13. Outbreak TB in a dormitory, with the same


10. A 53 y.o. farmer experiencing sub febris, source of infection, based on RFLP analysis. The
abdominal pain, diarrhea. In his skin there is a most appropriate health intervention?
black eschar surrounded with vesicle and there is a. Rehabilitation
edema near his mouth. A week ago he notices b. Prevention
that his cattle were sick, but his still eat half cook C. Curative
cattle meat. What is the organism? D. Medication
E. Environmental modification
a. Bacteroides fragilis

b. Leptospira interrogans
14. there is rna virus cause deathly outbreak of
c. Bacillus antrhacis hemoragic disease. Patient have feer, muscle
pain, headach, followed by abdominal pain. Rash
d. Pasteurilla pestis with internal n external bleeding. Hospital staff is
e. Borellia brugdorferi infected. Virus are highly virulent transmited by:

a. blood n body fluid

b. mosquito bite

11. a women and recently return from Lapung c. transmission to human from rodent excrata
complaining having paroxymal attack chill, fever d. aerosol transmission
and sweat. These attack last a day 2 days at time
e. ……….. b.dysentri

c. cholera

15. a 31 years old man with complaints of high d. giardia lambia


fever, cough, shortness of breath. 2 hours before
admission he experienced right sided chest pain e. entamoeba hystolitika
when took a deep breath and cough. Chest
radiography shows diffused bilateral interstitial
pulmonary infiltrate. Analysis of blood gas 18. Men, 44 tahun dari Jakarta pergi berbisnis ke
showed PO2= 60 and 91% HB saturation. Kalimantan selatan. Lalu disana dia pergi ke ER.
Hematology serum and liver are normal. He lives
near poultry. Patho agent are new viral. what are Sign and symptom = diare and abdominal
the prevention for prophylaxis? pain
a. vaccines PE = Pitting edema
abdominal and leg
b. Antitoxin
Lab culture = (-) enteric pathogen
c. Antibiotik
Blood = eosinophilia
d. Antiviral
Stool = large ellipsoidal,
e. Antidotes operkuled egg with thin and transparent sheet.
Tekena infeksi apakah bapak ini?

16. A 28 years old ,an comes withu high fever for Schistosoma Japonicum
8 days, malaise, headache and constipations. He Fasiolopsis Buski
denies any vaccines before. Temperature 39 C.
Blood pressure 130/90 pulse 72x/min. on Tricinella Spiralis
Physical examination there are
hepatosplenomegaly and faint erythomatous Echinococcus granulosus
measles.
Clornonsis sinonsis
what are the most likely pathogen?

a.vibrio cholera
19. bagaimana mekanisme terjadinya edema pada
b. Shigiella dysentry cerita diatas?

c. salmonella typhi a. host-allergic reaction because of the parasite

d. entero hystolytica b. the larva penetrate the wall

e. Giardia lambia c. hypersensitivity reaction because adult worm


in GI

d. there are accumulation of egg in GI


17. pasien 28 tahun demam 8 hari ada ciri2
sebagai berikut : erythematousmacula, myalgia, e……..
headache, vaksin tidak lengkap.

a. typhii
20. Pulang dari papua. %hari kemudian ada
intense chill fever. demamnya dengan always 1
day after. between those period, having low grade 24. edo…kerjasama nya please…. Demi kalian
fever, myalgia, nausea, vomitting. datang ke juga kok
klinik , 1jam yang lalu masuk ke ICU-agitated, ga
lama mati. Organism yang masuk ke host ini 25.  Komplikasi plg sring ???
berproliferation dmn? a. Pulmo hemoragik
a.brain B. m miopericarditis
b.liver
c.heart c. encephalitisa
d.spleen
d. Osteomyelitis

e. Anemia
21. 61yr old man,go to the neurolog clinic untuk
evaluasi peripheral neuropathy. He was found to
have dermal
hyperpigmentation/depigmentation(salt/pepper) 26. A 39 years old man, live in Manokwari
of the skin,scaly palms and transverse ridges on papua, developed a major seizure while at work,
hus fingernail which were identified as being a no history of epilepsy. Head MRI showed lesion
result of a chronic chemical intoxication. surrounding a scolex. The etiolohic agent would
What chemical agent? most possibly have been acquired by eat/drink:
A.acetaminophen A. Uncooked vegetables
B.arsenic B. Raw beef
C.cyanide C. Raw pork
D.flouride D. Uncooked fish
E.iron E. Unfiltered water

22. Chandra…kerjasama nya please…. Demi 27. anak cowo ada lesi di kulit seperti
kalian juga kok strawberry, gummata. Exam apa yang paling
cocok untuk diagnosis?

a. widal test
23. a 55 year old man from middle east had fever
and chill with weight loss, sweats, headache, b. IgM-IgG treponema
muscle pain, fatique, depression. The man is
c. dark field
daily farmer and a couple week before symptoms
appeared he drank glass unpasteurized raw milk. d. FTA-ABS
The culture of blood tiny gram (-) cocobacill,
catalase(+) and oxidase (+). Infected organism? e. ELISA
A. M. tuberculosis

B. S. typhii 28. A 27 year old man come with acute severe


encephalitis that require hospitalization. Several
C. Brucell species days before he got that disease he recognize
many pigs in his farm were died. Which of the
D. Pasteurella pestis
following is most appropriated natural host for
E. S. aureus the pathogen?
A. Arthropod
B. Horse b. copper deficiency
C. Bats
D. Squirel c. vitamin E deficiency
E. Rats
d. iron deficiency

e. folate deficiency
29. female tourist gastroenteritis ke Indonesia
makan Indonesia food. Abdominal cramp and
watery diarrhea. No fever or nausea or vomiting. 32. a lab examinaton of a leprosy patient after 4
Symptom resolve 24 hours no recurrent. months MDT therapy revealed bacterial index 6+
Pathogen? morphological index 50%. Which is your
a. s.typhi conclution for this result?

b. shigella dysentriae a. pausibacillar leprosy patient sensitive to


treatment
c. EHEC
b. multibacillar leprosy patient sensitive to
d. s. aureus treatment

e. ETEC c. multibacillar leprosy patient resistant to


treatment

d. pausibacillar leprosy patient resistant to


30. 23 years old male. 4 days history of fever, treatment
headache, retroorbital pain, myalgia and rash. PE
diffuse er e. pausibacillar leprosy patient prone to nerve
damage
resulting from a pressure applied to her skin.

morbili
33. A boy came with his mother to ER Siloam
dengue hospital
His mother said that before this, the boy just
influenza playing in the backyar.
PE: high fever 39'C and multiple stinging on the
coxackie back.
rhinovirus What is the most likely species that bite him?
A. Hymenoptera sp.
B. Dermatos andersoni
C. Tactodectus macta
31. a 40 year old lady came to the outpatient D. Loxoxceles laeta
clinic with a chief complaint of fatigue and E. Lytta Vesicatoria
difficulty in concentration for the last two weeks.
HT revealed that she had a balance diet, no
history of chronic disease and had been a heavy
smokers for 20 years. On PE it was found that she 34. A 28 years old lady came to Posyandu to have
had anaemic conjunctiva with other findings was a nutrition consultation. Last year, she delivered a
within normal limits. Lab showed that she had stillbirth baby with a spinal disorder. She and her
haemolytic anemia. What is the most likely cause husband start planning to have a baby again. She
for her condition? asked for suggestion about her dietary intake
before she got pregnant.
a. vitamin K deficiency
What kind of diet does she need to prevent the 38. 8 day infant born help of ----, irritable 2
same disorder for her future baby? days ago, noise cause pain episthotonus. PE
umbilical cord dried. Normal stimulated by
a. Dark green vegetables touch/ noise then begin to cry, stiff, arch its back.
b. Pasteurized milk Stiff continue until calm calm down. Diagnosis…
c. Deep water fishes
d. Boiled eggs Bacterial meningitis
e. Lean beef Botulism

General seizure
35. 27 yrs old women at term pregnancy had
fluctuated fever since 2 weeks ago. She looked Tetanus
pale, generalized weakness and headache.She
went to public health and was given Viral encephalitis
chloramphenicol 500mg 4x daily, paracetamol
500mg 3x daily. She is pregnanct and she didn't
tell. What is the side effect drugs will you find in
39. A married mom bring her 18 months son to
that baby?
ER due to rash that develop today. She reports
A. Reye syndrome that he has had a runny nose, conjunctivitis and
B. Gray syndrome diarrhea. On PE he looks ill, temperature 40oC
C. Down syndrome and diffuse 2-3 mm erythematous rash begin
D. ADHD from hair line behind ear and spreading to face
E. Black water fever and body. On buccal mucosa, red spot are noted.

What is the most likely causing agent?

36. Soalnya ada orang yang mau ke luar dari a. Rubella


jawa dia mau ke papua nah disitu ada banyak b. Coxsackie virus
plasmodium tratment apa buat mencegah c. Morbili
penyakit tersebut d. HSV-6
Jawabannya: doxycycline e. Parvovirus B19

.40 : most severe complication from this disease


37. 40 years old man come to hospital with 4 (morbili)
times generalized tonic clonic seizure since 2           a. osteomyelitis
weeks ago. No history of seizure & head trauma.           b. encephalitis
1 year ago, had bussined in papua for 6 oths.           c. otitis media
During his stay, he ofte osumed undercook pork.           d. sinusitis
Brain MRI show small cyst??           e. oral thrush
a. taenia eggs

b. cysticercus cellulose 41. 48 years old acute gastroenteritis has step


c. t. solium worm nadder night fever since 7 days ago. He also has
abdominal pain and constipation. Biasa makan
d. t.saginata worm uncooked vege at streets . Which is most likely
organism?
A. Vi antigen
B. Urease
C. Hemolysin
D. Shiga toxin B) plasmodium ovale.                             
E. Pili
C) plasmodium malariae.                       

D) plasmodium falciparum.                     
42. Seseorang mengalami sore throat, ada grayish
white di ujung mulut, limfadenopati. E) plasmodium menangle
A. Dipteri
B. Pertusis
C. Tularemia 46. Antidotum untuk keracunan analgesic-
D. .... antipyretic…
E. ....
a. Amylnitrite
b. Atropine
43. Baby rash identically, mother had febrile c. BAL
illness during the second trimester of pregnancy. d. EDTA
Examination reveals diffuse raised purple skin e. N-methyl cystein
lesion. There is no pallor, jaundice or cyanosis.
The baby has cataract a 3/6 systolic heart
murmur. Enlargement of both liver and spleen, no 47. man 47 years old come to your clinic. he has
lymphadenopathy. complain about malaise,high fever,and mialgia,
slightly jaundice.one week ago he has clean a
a. Congenital toxoplasma canal with his friend..apa yang harus kamu
b. Congenital rubella lakukan sebagai dokter untuk profilaksis dalam
c. Congenital cytomegalo kondisi demikian>>
d. Congenital herpes simplex
a. antiviral,
e. Congenital varicella
b. antibiotik
44. 40 years old woman, presented with c.antitoxin
raised redness plaque resemble doughnut with
raised border that she already had for 1 year, no d. vaksin
itchy, weakness on her leg. Her slipper left.
Annular erythematous sharp border, shiny
surface, enlarge popliteal nerve. Lab exam?
48. women come back from Saudi Arabia, has
a. KOH weakness and tingling sensation in her right leg,
b. Zielh nelson other neurology exam is normal including her left
leg..Which immunization did she miss?
c. Gram
d. Na cl a.MMR

b.DPT
45. A girl returned from Lampung with
paroxysmal fever, malaise, and myalgia. The e.Oral  Sabin
fever recurrs every 36 to 48 hour. On microscopic
examination there are ring and cresecent like
shapes inside the red blood cells. This is
49. Seorang anak dengan fever, konjungtivitis,
characteristic of?                 
rash dari line belakang rambut kepala sudah sakit
A) plasmodium vivax.                              sejak 2-3 hari lalu dari genus apakah itu??
Morbili
D. Delayed reaction
50. Severe complicationnya apa? E. Quantal reaction
Encephalitis
Pilihan yg lain g lupa
55. male, 46 y.o came to you as a doctor with
complaint 'sesak nafas' in
51. 34 yo women, G2 delivers at 36 week of his work place.
growth-restricted infant w/ catarract, PDA, and but when he back to home, 'sesak nafas' is
sensorineural deaf. Mom had flu-like syndrome dissapear. he notice, that
with rash in early pregnancy. Etiology? his sister using broncodilator. he also complaint
A. Porvavirus sneezing in morning.
B. Rubella he work in sandbusting company in last 2 week.
C. Morbili From the following what
D. CMV the appropriate measurement for hazardous thing.
E. HSV a. Body mass index
b. nutrition
52. 27 yo woman, abis travelling southeast asia. c. activity susceptibility
Shaking, chills, 40C. Ada P.vivax. Cr treatment d. -
hypnozoit? e. tissue and organ
A. Primaquine
B. Pyrimethamine
C. Quinine
D. Chloroquine 56. Anak Kecil perempuan berusia 3th
E. Chologuanide mengeluh tidak mau makan, badannya kecil,
terdapat anemia, pada pemeriksaan fisik terdapat
hepatomegaly, pada pemeriksaan lab terdapat
eosiofilia, kata ibunya dirumahnya terdapat
53. Woman 25 y.o, G3 at 39 weeks delivers a anjing dan dia suka main dengan anjing tersebut.
small gestational age infant with chorioretinitis,
intracranial calcification, micro-cephaly, Kemungkinan gejala yang terjadi
jaundice, hepatospleenomegaly. The nurse
reports poor feeding and tone. Mother denies any a. Creeping Eruption
raw or undercooked meat, no cats. She works as b. Biliary Cirrhosis
nurse in pediatric ICU. What is the most likely
causative agents? c. Cholecystitis
A. CMV
B. Epstain Barr virus d. Pruritus Ani
C. Hepatitis B virus
D. Parvovirus e. Visceral Larva Migrans
E. Yellow-fever virus

54. 45 y.o male has a chronic skin lesion both 57. Anak kecil datang ke klinik dengan
hands for 1 year. Skin erythematous, crusted, keluhan gatal2 pada pantatnya, ditemukan
itching, and have serous discharge. Work in a sesuatu seperti kuku, diambil dan didapatkan
gold refining plant for 2 years without protection. telur berdinding radial
Contact with cyanide. Not wearing gloves. What
is the possible mechanism? Penyebabnya..
A. Immediate reaction
B. Acute reaction a. Taeniasis Saginata
C. Gradual reaction
b. Taeniasis Solium
c. Merozoite discharged from schizont and enter the
blood
d. Oxyuris
Process of gametocyte in blood
e. Ascariasis
Pigment in the parasite is formed

58. Laki2 terkena recurrent fungal infection di


bagian kaki dan gatal2. Dia ke dokter dan telah 61. Symptoms : Fever , nausea, fatigue. Back
dikasih obat. Dia minta advice agar penyakit from Kalimantan. Blood smear revealed schizont
tersebut tdk terulang. Apa yg anda sarankan? isi merozoit 12-24, erythrocyte enlarges. The
pathogen is :
A. Hindari makan seafood a.VivaxMalaria
b.FalciparumMalaria
B. Selalu pake sandal. c. MalariaeMalaria
C. Jaga tetap kering dan bersih. d. OvaleMalaria
e. DowleskiMalaria
D. Selalu pake sepatu
62. 52 years old man is brought to the emergency
E. Jangan digaruk department in an unconscious condition on
examination, he was death form more than one
hour. On autopsy of the brain, there is cyst’s fluid
containing protoscoceles with hooklet and many
A 30 year old woman, presents with abdominal
daughter cysts were found. What is the most
pain and diarrhea of 3 days duration. She doesn't
likely diagnosis of the disease?
complaint of nausea, vommiting or fever. She has
a. Hydatidosis
no sick contact or significant travel history. A
b. Cysticercosis
stool sample is obtained, which reveals
c.Serebralmalaria
rhabditiform larva.
d. Histoplasmosis
e.  Toxoplasmosis
59. In condition below adult form of the
pathogenic agent can be found in the whole GIT
and its larvae can be found in visceral organ
(lung, liver, gallbladder) is : 63. dewi…kerjasama nya please…. Demi kalian
a. eosinophilia juga kok
b. hyperinfection
c. retroinfection 64. dewi…kerjasama nya please…. Demi kalian
d. hypereosinophilia juga kok
e. autoinfection

65. Pengambilan specimen untuk edema pada leg


60. a woman came with the complaint of being dan scrotum pada filariasis adalah terbaik pada
uncomfortable and hardened in the left upper side waktu….
abdomen. Every 4 days, she is intermittently
feverish, on blood smear exam, plasmodium Answer: 10 – 12 pm
parasite (+). Fever symptoms on previous
referred to:
66. livia…kerjasama nya please…. Demi kalian
Sporozoit in blood juga kok
Hypnozoit in liver is found
67.livia…kerjasama nya please…. Demi kalian 73. 34 years old women complain sudden onset
juga kok high fever 4 days with nausea, vomite, muscle
headache, headache, pretechie + . which of the
68. lauren…kerjasama nya please…. Demi kalian following laboratorium result is the most
juga kok appropriate with the patiet diagnosis?
69. kesalahan banker… maap yei.. a. trombositopenia

b. neutropenia
70. 35 years old male, farmer who was working
with pestiside  was taken to ER with headache, c. anemia
vomiting, salivation, diarrhea, muscle
fasciculation, difficulty walking, difficult d. hematocrit decrease
speaking. His cloths has been removed, hes been
washed, has been given activated charcoal. What e. leukosit decrease
is the most effective remains treatement for the
case of pesticide poisoning?
74. 1 years old baby get swollen gum 1 week. On
a. Epinephrine history, he has normal body weight, breast feed
b. Antacid until now, balance diet with high intake cooked
c. Spironoloctate vegetable and fruits since 6 month old. No history
d. Atropine of cronic disease. Whats the most likely cause of
e. HCT the complaining??

a. protein deficiency because of breastfeed

b. Karbohidrat deficiency because of increase


71. A 65 year old woman suddenly had flaccid kalori need
paralysis at legs. Few days ago, there are
headache, nausea, fever, sore throat. Her son: c. fat soluble vitamin deficiency because of high
daughter in law, and young grandchild have vegetable and fruit intake
vaccine routine oral.. She taking
d. water soluble vitamin deficiency because of
imunosuppressant for kidney transplant. Vital
cooking
sign, cranial nerve, CT-scan and MRI normal..
The transmission? e. mineral deficiency because of competition with
A. Droplet vitamin absorbtion
B. Airborne
C.fecal oral
D. Close contact
E.animal bite 75.Male 10 year old,high fever 38 C,runny
nose,sore throat,mild cough, vesicular lesion at
72. Pork eating village in papua new guinea palatum,dorsal,n palmar of the hand. Most likely
epidemic outbreak of epilecticum seizure. The diagnosis:
investigate: A.adenovirus
A. Level of balantidium coli in swine stool. B.coxsackie virus (ini jwbnnya)
B. Practice of consuming raw decease human C.
brain D.sinusitis
C. Presence taenia eggs in drinking water E.pneumoni
D.presence of trophozoite in human blood
E. Quantity of culidare in the village
76. 19 years old girl come with sign and skizon 12 – 24, merozoit 20, RBC
symptomp of poisoning. Nausea, vomiting, enlarges. The cause of the fever:
abdominal cramp, diare, excessive saliva, a. Sporozoit in blood
headache, giddiness, rinorhea, pin point pupil, b. Hypnozoit in liver is found
and mental confusion. What ‘s the most possible c. Merozoite discharged from
agent? schizont and enter the blood
d. Process of gametocyte in blood
a. acetaminophen e. Pigment in the parasite is formed
2. 35 years old male, farmer who was
b. barbiturat working with pestiside was taken to ER
c. carbamate with headache, vomiting, salivation,
diarrhea, muscle fasciculation, difficulty
d. sianida walking, difficult speaking. His cloths
has been removed, hes been washed, has
e. opiate been given activated charcoal. What is
the most effective remains treatement for
the case of pesticide poisoning?
a. Epinephrine
77. 22 years old women, G1, complaining flu like
b. Antacid
illness during third trismester early. She delivery
c. Spironoloctate
a growth restricted infant with seizure intracranial
d. Atropine
calcification, jaundice. Whats the most possible
e. HCT
cause?
3. Man, 31 years old, inguinal
a. treponema pallidum lymphadenopathy, blister on penis, had
unprotected sex 1 month ago.
b. T.vaginalis Penyebabnya apa?
a. HPV
c. toxocara sp b. Triponema Pallidum
c. Haemophillus Influenzae
d. neisheria meningitis d. Chlamydia trachomatis
e. HSV
e. toxoplasma gondii
4. 4 years old, child sleeping scratching
anal. Suspect pin worms. BEST diagnosis
is?
78. Influenza virus a. Stools for eggs
b. Stools for larva
c. Blood smear of microfilm
d. Transparent adhesive tape of
79. ETEC eggs
e. Titer IgE antibody
5. Clotted tongue, watery diarrhea, demam
SOAL UJIAN tinggi pas malem
a. Salmonella
TROPICAL MEDICINE BLOCK b. E. Coli
c. Malaria
d. …
e. ..
1. 40 tahun man, fever  1 minggu. Ada 6. Best treatment for scabies
nausea, fatigue. Back from Timika. a. Sulfur
Blood smear revealed late trofozoit, b. Gamexane
c. Crotamiton
d. Lindane c. Naloxone
e. Permethrine d. Nitrite
7. 31 years old female underwent allogenic e. Protamin
bone marrow transplant for AML. 12. 30 years old male, having fever for 3
Second month after, high fever, non days, with falciparum rings +++. He was
productive cough, breathlessness, given CQ3 ( chloroquine for 3 days),
splenomegaly. Lab : anemia, leucopenia, after 3 days, the patients still feverish,
thrombositopenia, abnormal liver blood smear examination show
function test. X-ray: diffuse interstitial falciparum rings ++++. What is the
pulmonary infiltrates. Bronchoalveolar possible respon of CQ?
samnple performed. Infected kidney cells a. Sensitivitiy
 owl eye. Best treatment: b. R1 resistance, early
a. Anti microbial recrudescence
b. Anti viral c. R1 resistance, delayed
c. Anti toxin recrudescence
d. Immunosuppresant d. RII resistance
e. Anti fungal e. RIII resistance
8. this female illness could be most easily 13. Refer to the previous case above, what is
diagnosed by which of the following the treatment of choice
test? a. Primakuin
a. HSV – 1 IgM Ab b. Amodiakuin
b. CMV antigenemia c. Meflokuin
c. HHV – 6 IgM Ab d. Sulfadoxine – pyrimethamine
d. Culture of blood in erythroblastic e. Artemisin
cell 14. 24 tahun female, myalgia and weakness
e. Monospot – test of her right leg. Dari Saudi Arabia,
9. One member of the NGO team from imunisasi ga komplit. Normal sensation
Indonesia has been reported dead in on right leg, movement and cranial nerve
Mexico, because of spider’s bite. The normal. Which is a target cell receptor
possible cause of this disorder is: for this pathogenic agent?
a. Erucism a. Silica Acid on mucous cell
b. Lepidopterism b. Ach on neurons
c. Tick Paralysis c. CD 4 on T lymphocyte
d. Arachnidism d. C2d complement on B
e. Delusional Parasitosis lymphocyte
10. Ulcer + lethargy + pale + face = thin old e. Ig on epithelial cell
person + rib prominent 15. Which one of the following is the best
a. Marasmus specimen for diagnosing in case above?
b. Kwashiorkor a. LCS
c. Marasmic Kwashiorkor b. Blood
d. Energy malnutrition c. Urine
e. Protein malnutrition d. Throat swab
11. 46 tahun female, several bunion e. Rectal swab
removed from her right foot. Chooses 16. Young boy, high fever, difficulty
conscious sedation other than general  swallowing water, hydrophobia,
become agitated, combative, given IV disorientation, hypoesthesia. In the left
midazolam. Drug that counteract arm, 4 weeks ago was bitten by dog.
midozolam? (Keracunan obat anesthesia Characteristic of pathogenic : zoonotic,
midazolam di kasih apa?) single stranded, negative sense DNA
a. Flumazenil virus. Which do you found for diagnosis?
b. Glucagon a. Giant cell
b. Limfosit plasma biru e. Hepatitis B, Hepatitis B, pertusis,
c. Negri bodies poliomyelitis
d. Inclusion bodies 21. Keracunan acetaminophen, kasih apa?
e. Clue cell a. Acetylcysteine
17. Anak laki laki 4 tahun dating dengan b. ..
pucat dan intense chills, daily high fever. c. ..
Memiliki sickle cell disease. Platelet d. ..
53.000, WBC 17.000. Description the e. ..
relevant finding? 22. 45 years old man suffered otitis media 2
a. .. years ago. He has headache a week ago.
b. .. Radiographic exam shows 3 cm round
c. .. mass.
d. . a. Neoplasm
e. .. b. Aneurisma
18. female 27 tahun, fever 2 hari, c. Infarct
unconscious and general seizure 2 hour d. Abscess
ago. Physical Examination : GCS 3, BP e. Hemorrhage
90/60 mmHg, temperature 41 C, Hb 10.3, 23. Appropriate antiepileptic drug for
WBC 5800, platelet 39.000. blood smear Absence Seizure?
ada schizont. State yang di takutin setelah a. Carbamazepine
ini? b. Valproic Acid
a. … c. Benzodiazepine
b. Intermittent fever d. Lumotrigine
c. Gametocyte formation takes e. Ethuxusimide
place in visceral organ 24. 52 tahun man is brought to the
d. Make capillary destruction in emergency department in an unconscious
CNS by parasit contain condition on examination, he was death
erythrocyte form more than one houn on autopsy of
e. There are many drugs resist once the brain, there is cyst’s fluid containing
aginst P.falciparum protoscoceles with hooklet and many
19. A one year old boy was diagnosed with daughter cysts were found. What is the
measles since a week ago. This morning, most likely diagnosis of the disease?
his mother noticed that he also suffered a. Hydatidosis
for swollen, reddish gum that signified as b. Cysticercosis
scurvy. Most likely mechanism that c. Serebral malaria
underlines his recent condition? d. Histoplasmosis
a. Impaired absorption of e. Toxoplasmosis
micronutrients 25. 5 year old girl , 2 hari fever + throat.
b. Altered micronutrients Ada lethargy , grayish white membrane
metabolism di faring, tonsil asimetris, treatment?
c. Depletion of micronutrient stores a. Abx, antiviral
d. Micronutrient redistribution b. Antiviral, antifungi
e. Negative mincronutrient balance c. Antitoxin, antifungal
20. Group III tropical disease d. Abx, antitoxin
a. Leprosy, lymphasis filariasis e. Antiviral, antitoxin
b. Malaria, TB, Dengue, 26. Shortness of breath at work place. At
Schistosomiasis home, symptoms missing. Sneeze every
c. Ebola, Avian Influenza, SARS morning. Worked at sandblasting
d. Protein calory malnutrition, company and expose to dust since 2
pesticide intoxication weeks ago. Possible cause:
a. Wood
b. Quartz 32. Female tourist developed gastroenteritis
c. Tobacco smoke and tasted some Indonesian food. Onset
d. Coal abrupt with abdominal discomfort and
e. Ammonia watery diarrhea. Setelah di periksa,
27. doctor gave him sick leave certificate 2 ditemukan pathogen, yaitu…..
days and asks the patient to come back a. Salmonella typhii
tomorrow to spirometry. Result of b. Shigella dysentriae
spirometry do you expect? c. Enterohemorrhagic coli
a. FEV1 turun d. Enterotoxigenic colli
b. FVC turun e. Staphylococcus aureus
c. FEV1 turun, FVC turun 33. Seorang pria dari bogor akan pergi ke
d. Normal samarinda selama 3 minggu, dikasih obat
e. MVV naik apa?
28. Seorang bapak tinggal di Lembah Napu a. CQ + PQ
di Sulawesi, terjadi perbesaran hati, b. Sulfadoxine + PQ
flatulence c. Aretusunate + AQ
a. Schistosoma japonicum d. Kina + Tetracycline
b. Schistosoma mansoni e. Doxycycline
c. Schistosoma haemotobium 34. 65 tahun female, tiba tiba lumpuh kaki
d. Schistosoma rodentisia kiri. Beberapa hari lalu, sakit kepala,
e. … fever, sore throat, nausea. Tinggal ama
29. Cewe 18 tahun come to obgyn dengan no anak laki lakunya, menantu perempuan,
menses 2 bulan, lose weight 5 kg for 3 dan cucu yang masih kecil yang baru
bulan. Lagi dalam proses pengobatan TB dapet imunisasi rutin, si nenek itu pake
4 bulan yang lalu immunosuppressant untuk transplant
a. Increase glukoneogenesis ginjal. Vital bagus, cranial nerves ok.
b. Increase vitamin excretion Head CT normal, Lumbar MRI normal.
c. Increase resting metabolism Cara transmisi?
d. Increase CHO store a. Droplet
e. Increase protein catabolism b. Airborne
30. 70 tahun cowo ke ER gara gara seizure, c. Fecal-oral
Istrinya jam 5 pagi bangun, lantaran d. Close contact
suaminya seizure. Kepalanya deviasi ke e. Animal bite
kiri, tangannya kenceng, trus ada 35. 35 tahun male, seizure, CSF = neutrofil,
generalized body jerking + RBC, amoeba form on wet microscopy
unconsciousness selama 2 menit, stop a. Naegria fowleri
spontaneously. No history of seizure. He b. Entamoeba histolytica
is drowsy and confused. BP 140/195, HR c. Acanthamoeba costelorii
96, RR 20, susuh 36.5 C, Resting Blood d. Escherichia colli
Glucose 610. e. Plasmodium falciparum
a. Pseudoseizure 36. 22 tahun, female, 2 week di Papua, blood
b. Primary epilepsy and slimy diarrhea dari lasat 5 days in
c. Secondary epilepsy Papua. > 5 kali bowel movement each
d. Acute provoked seizure day, stomach cramp, pain stool, no fever,
e. Generalized seizure abdomen tender. Agak dehidrasi
a. E.histolytica
b. Cholera
31. Treatment  Insulin cutaneous c. Rotavirus
d. Salmonella
e. E.colli
37. Tidak inget udah imunisasi TT atau the following, which parasite can cause
belum, prophylaxis nya apa? this?
38. Seizure karena tetanus disebabkan oleh? a. P. falciparum
39. 3 years old boy, brought to emergency b. P.vivax
unit in Tangerang hospital with stridor, c. P.malariae
dyspnea, and croupy cough. What is most d. P.ovale
likely pathogenic agent that cause the e. ……
patient’s disease? 47. Lab test buat pertusis?
a. B. pertusis 48. 10 tahun, diarea, blood and mucus, rectal
b. H. influenzae prolapsus
c. Coxsavirus a. Ascaris
d. B. paraapertusis b. Necator
e. C. diphtheriae c. Ancylostoma
40. Keracunan parathion  d. Trichius
acteylcholinesterase e. Enterobius
41. 2 tahun, febrile, deteriorate, worsening 49. parasit ini tinggal di?
cough and dyspnea. Child had obvious a. Liver
respiratory distress with retraction and b. Lung
elevated respi rate. X-ray chest c. Small bowel
hiperinfiltration, both lundg field. What d. Stomach
is the lab? e. Caecum
a. Nasopharyngeal washing 50. Apa yang bisa bunuh virus rabies?
b. Trache aspirate a. Infrared radiation
c. Bronchial washing b. Catalase
d. Nasal swab c. Alkali water
e. Blood d. Sunlight
42. At nite, 28 years old, male, right foot e. Heating at 60 C for 30 minute
bitten by animal 1 hour before. Heard the
sound of the animal. Pain in Right foot, 2
bitten mark, edema, erythema, in dorsal CUCU
right foot. What likely animal?
43. Initial management? (HENDRA-NIPAH VIRUS)
a. Obvservation 1. A 27-year-old man develops acute severe
b. Cryotherapy encephalitis that requires hospitalization.
c. Antivenom Several days before he got that disease, he
d. Inicision and suction recognizes that many pigs in his farm were
e. Corticosteroid died.
44. Next step: Which of the following is most appropriated
a. Wide excision natural host for the pathogen?
b. Fasciotomy f. Arthropod
c. Antivenin g. Horse
d. Incision and suction h. Fruit bats
e. Corticosteroid i. Squirrel
45. after 12 hours, severe pain especially j. Rats
more right leg and leg pallor, cold, no
pulse in R dorsal artery. Treatement??? 2. A 32-year-old female developed a “flu-like”
46. a woman came with the complaint of syndrome with high fever up to 400C,
being uncomfortable and hardened in the anorexia, headache, and myalgia. Four days
left upper side abdomen. Every 4 days, later, she become confused and agitated with
she is intermittently feverish, on blood difficult to breath. Chest x-ray showed
smear exam, plasmodium parasite (+), of parenchymal infiltrate. One of her kids has
also these symptoms, and has already passed 5. What antibiotic should the doctor give to
away. In order to provide laboratory treat the worker?
conformation of the pathogenic agent, a f. Tetracycline
tissue culture was ordered. g. Aminoglycoside
Which of the following would be the best h. Ciprofloxacin
specimen for isolating the pathogenic agent i. Neomycin
responsible for this infection? j. Crystalline penicillin G
f. Stool
g. Blood 6. Which of the following virulence factors is
h. Saliva most likely to be involved in the pathogenesis
i. Lung biopsi of illness?
j. Nasopharyngeal swab f. Exotoxin
g. Endotoxin
Question 3-4 are linked to the following case: h. α-hemolysin
(RABIES) i. Lipopolysaccharide
3. A young man was admitted to hospital after j. Antiphagotic factors
increasing left arm pain and paresthesia.
Several days ago, he contacted with a (ANTHRAX-GI)
dog……. His symptoms increased and were 7. A 53-year-old male farmer developed low-
accompanied by hand spasm and sweating on grade fever, abdominal pain, and diarrhea. He
the right side of the face and trunk. This has a skin lesion of black eschar surrounded
patient was admitted to the hospital the day by vesicle and edema near his mouth. A week
after developing dysphagia, hydrophobia, before, his notice many cattle in his farm
hypersalivation, and disorientation. were sick, but he still eats a half-done beef
Which of the following is usefull for making steak from his cattle.
a diagnosis in this case ? What is the following microorganism is most
f. Giant cell likely involved in this case?
g. Limfosit plasma biru f. Bacteroides fragillis
h. Negri bodies g. Leptospira interrogans
i. Inclusion bodies h. Bacillus anthracis
j. Clue cell i. Pasteurella pestis
j. Borrelia burgdorferi
4. Which of the following is available for
treating this patient? Question 8-8 are linked to the following case:
a. Immune globulin (EMERGING AND RE-EMERGING
b. Live attenuated vaccine INFECTIOUS DISEASE)
c. Antiviral There is an RNA virus that causes deadly
d. Antimicrobial outbreaks of hemorrhagic disease in Africa. In
e. Antitoxin each outbreak, hospital staff became infected.
This virus is highly virulent.

8. Which of the following is the most likely


Question 5-6 are linked to the following case: transmission for this disease?
(ANTRAX-CUTANEOUS) f. Spread by contact with blood or other
A 32-year-old worker from a farm comes with a body fluid
painless dermal papule on his right hand which g. Transmitted by mosquitoes bite
started to develop since five days ago. The h. Transmitted to human from rodent
worker says as the lesion getting bigger, and the excreta
skin becomes black. He has tender axillaries i. Aerosol transmission
lymph node enlargement. j. Direct contact with cattle product
e. Tsunami impact

13. An 80-year-old man is developing fever,


cough, runny nose, headache, and myalgia.
9. Which virus is most likely the cause of this He lives in a nursing home. Many of people
outbreak? in his neighborhood have this disease.
a. Dengue virus Which immediate course of action would be
b. Menangle virus most appropriate for this patient?
c. Lassa virus a. Vaccination
d. Ebola virus b. Immunoglobulin
e. Influenza virus c. Rimantadine
d. Acyclovir
10. High priority bioterrorism agents include e. Cytokine
organisms that pose a risk to national security
because they can be easily disseminated or 14. A 22-year-old man who works in the
transmitted from person to person. Which of pediatric ward of a hospital suffers from
the following is most likely to be transmitted malaise, sneezing, and runny nose. He
from person to person in bioterrorism act? subsequently develops a mild sore throat,
a. Bacillus anthracis headache, and stuffy nose. The symptoms
b. Clostridium botulinum resolve within 4 days. Which virus is most
c. Francisella tularensis likely to be responsible for these symptoms?
d. Variola major f. Rota virus
e. Brucella melitensis g. Rubella virus
h. Coxsackie virus
Question 11-12 are linked to the following case: i. Hepadnavirus
(AVIAN FLU) j. Influenza virus
A 31-year-old man presented with complaints of
high fever, cough, and shortness of breath. Two (TIFOID)
ours before admission, he experienced right-sided 15. A 48-year-old man with acute gastroenteritis
chest pain when took a deep breath or coughed. has sub febrile fever since 7 days ago. He
Chest film showed diffused bilateral interstitial also has abdominal pain, and diarrhea. He
pulmonary infiltrate. Arterial blood gases showed usually eats undercooked egg every morning.
a PO2 of 60 mmHg with 91% hemoglobin Which of the following is most likely to be a
saturation. The hematologic, serum and liver constituent of this organism?
tests were normal. He lives near the poultry, and f. Vi antigen
the pathogenic agent is a new virus. g. Urease
h. Hemolysin
11. Which of the following is the prevention for i. Shiga toxin
this disease? j. Pili
f. Vaccine
g. Antitoxin 16. An outbreak investigation note many
h. Antibiotic costumer from a café were admitted to the
i. Antiviral hospital. They have fever; nausea-vomiting,
j. Antidotes constipated or diarrhea, weakness and altered
mental status. Rose spots are seen on the
12. What is the following is probably the reason trunk. Blood cultures from patients grow a
for the emergence of this infectious disease? non-lactose-fermenting gram-negative rod.
a. Animal husbandry practice In which of the following sites is a bacterium
b. Genetically engineering most likely to be found?
c. Vector elimination f. Blood
d. Antibiotic resistance g. Kidney
h. Liver g. Liver
i. Intestine h. Brain
j. Gall bladder i. Spleen
j. Renal
Question 17-18 are linked to the following case:
21. What is the most common mode of
A 28-year old man presented with 6-day history transmission of this agent?
of increasing fever, malaise, headache, and a. Contact with rat’s of urin
constipation. He did not receive any prior b. Transplacental crossing
vaccinations. His vital signs revealed bradycardia c. Blood transfusing
and fever of 400C. His physical examination d. Bite of a mosquito
revealed mild hepatospleenomegaly and faint e. Eat the contaminated food
erythematous macules.
22. An apparently run-down but alert 34-year-old
17. Which of the following is most likely to have woman comes to your office after 6 months
caused this man’s illness? spent as a teacher in a rural West Sumba. Her
f. Vibrio cholerae chief complaints are frequent headaches,
g. Shigella dysentriae occasionally nausea and vomiting, and
h. Salmonella typhi periodic fever. To rule out your differential
i. Entamoeba histolitica diagnosis, a smear of finger-stick blood was
j. Giardia lambia done. Which of the following choices would
fit your diagnosis based on your microscopic
examination of the blood smear?
f. Schizonts in red cells with 8-12 progeny
g. Rounded gametocytes present
h. Enlarged, somewhat misshapen
parasitized red cells
18. Which of the following specimen is most i. Numerous large ovoid parasites in some
appropriate for case above? of the red cells
f. Feces j. Numerous band-shaped tropozoites in the
g. Blood parasitized red cells
h. Urine
i. Throat swab
j. Biopsy

Question 18-20 are linked to the following case:


(MALARIA)
A 24-year-old man returned from Papua Island.
Five days later, he developed repeating intense
chills and high fevers. These severe episodes of (NEUROCYSTICERCOSIS)
fever had been occurring every other day. In 23. A 39-year-old woman live in Manokwari,
between these episodes, he had low-grade fever, Papua Island developed a major seizure while
myalgia, nausea, vomiting, and diarrhea. A few at work. She had no history of epileptic
hours ago, he was admitted to ICU in coma. He disease. A head MRI was remarkable for a
became progressively somnolent and died a week lesion surrounding a scolex. The etiologic
later. agent would most plausibly have been
acquired by eating or dinking which of the
20. Which of the following organs does this following food items?
infectious agent initially proliferate after f. Uncooked vegetables
entry in the infected host? g. Raw oysters
f. Heart h. Raw pork
i. Uncooked fish d. ELIZA assay
j. Unfiltered water e. PCR

24. A pork-eating village in the highlands of


Papua New Guinea is reported to be suffering 27. How did the man get infected form this
from an epidemic outbreak of epileptiform disease?
seizures. You have been sent to investigate. a. Ingestion of infected unpasteurized cow
One of the first things you should investigate milk
is? b. Needle stick injury from a contaminated
f. The level of Balantidium coli in swine syringe
stool c. Direct contact with infected animal parts
g. The practice of consuming raw human through intact skin
brains from recently deceased family d. Aedes aegypty bites
members e. Fresh orange juice consumption
h. The presence of taenia eggs in the
drinking water Question 28-29 are linked to the following case:
i. The presence of tropozoites in the human (TULAREMIA)
blood A 55-year-old warden found a dead muskrat on
j. The quantity of culidae in the village the bank of a stream. He picks up the animal and
buried it. Four days later, he developed a 1.5 cm
painful ulcer on the index finger of his right arm,
dr. ANGELA a 1 cm ulcer on his right forehead, and pain in his
right axilla. Physical examination also revealed
Question 25-27 are linked to the following case: right axillary lymphadenopathy.
(BRUCELLOSIS)
A 45-year-old man from the Middle East had 28. Which of the following is working diagnosis
fever and chills, with weight loss, sweats, for this patient?
headache, muscle pain, fatigue, and depression. f. Brucellosis
From physical examination the doctor found g. Pertusis
lymphadenopathy and spleenomegaly. The man h. Leptospirosis
is a daily farmer and a couple weeks before the i. Tularemia
symptoms appeared he drank a glass of j. Hydatidosis
unpasteurized cow milk. The culture of blood
grew a tiny Gram-negative coccobacilus, catalase 29. Which of the following prevention should be
and oxidase-positive; that resemble fine grains of done for high risk persons, such as research
sand. laboratory personnel from this disease?
a. Education
25. The patient was probably infected with which b. Immunization
of the following microorganism? c. Laboratory precaution
f. Mycobcaterium tuberculosis d. Burning infected animal
g. Salmonella typhi e. Antibiotics taken
h. Brucella species
i. Pasteurella pestis Question 30-33 are linked to the following case:
j. Staphylococcus aureus (LEPTOSPIROSIS)
A volunteer man returning from a tsunami area
26. Which of the following diagnostic laboratory was admitted to hospital. A week before, he had
test should be done for identification of this fever and headache. These symptoms resolved,
fastidious coccobacilus? but the day before admission he became pyrexial
a. Fluorescent antibody test and on examination was found to be jaundice and
b. Western Blot to have an elevated blood urea. Urine was
c. Culture collected and inoculated into a semisolid agar
medium, and examined by dark ground a. Testing serum using the rapid plasma
microscopy. reagin (RPR) test
b. Culture the urine on human diploid
30. Which of the following is your working fibroblast cells
diagnosis? c. Testing serum by darkfield microscopy
f. Syphillis d. Testing serum for antileptospiral
g. Pertusis antibody
h. Weil’s disease e. Culture of CFS on chocolate agar
i. Lyme disease
j. Relapsing fever 35. Which of the following is the prevention after
exposure?
f. Human vaccine
g. Health education
h. Vector controll
i. Isolation the infected people
31. Which of the following disease is the j. Antibiotics prophylaxis
complication of this disease?
f. Pulmonary hemorrhage
g. Myopericarditis
h. Aseptic meningitis
i. Osteomyelitis
j. Anemia
Question 36-38 are linked to the following case:
32. Which of the following reservoir animal is (YAWS)
the source of human infection? A young boy had an ulcerating papule on his
a. Fish legs. Later, this lesion destroys the bone of his
b. Bird legs (gummata), but three are no visceral and
c. Reptile nervous system complications. The disease is still
d. Scallop endemic in hot tropical countries, such as
e. Rat Indonesia.

33. Which of the following is drug of choice of 36. Which of the following microorganism is the
this disease? cause of the disease above?
a. Penicillin G a. Treponema pertenue
b. Chloramphenicol b. Borrelia burgdorferi
c. Erythromycin c. Leptospira interrogans
d. Gentamycin d. Spirillum minor
e. Rifampin e. Chlamydia trachomatis

Question 34-35 are linked to the following case: 37. Which of the following drugs is the drug of
(LEPTOSPIROSIS) choice for the disease above?
A 27-year-old medical student was admitted to a. Tetracycline
hospital because of sudden onset fever up to 390C b. Penicillin
and headache. Two weeks previously he c. Doxyccycline
volunteers cleaning the canal with others. Blood d. Vancomycin
tests done shortly after admission indicated renal e. Clarithromycin
function abnormality and elevated liver function
tests. On examination, he looked so yellow. 38. Which of the following examination is the
most appropriate to diagnosis the disease
34. Which of the following would be most likely above?
to confirm the diagnosis? f. Widal test
g. IgG and IgM of Treponema j. Cholera and ETEC vaccine
h. Dark-field microscope
i. FTA-ABS test and TP-PA test dr. SANDRA
j. ELIZA assay (DHF)

42. A 23-year-old man sees his family physician


dr. ROBERT TEDJA with a sudden onset of 4-day history of
fevers, headache, retro-orbital pain, myalgia
Question 39-40 are linked to the following case: and rash. Physical examination shows diffuse
(TRAVELER DIARRHEA AND THE erythroderma with blanching erythema and
PREVENTION) petechial formation resulting from pressure
A female tourist developed gastroenteritis while applied to her skin. Laboratory test reveal
visiting small town in Indonesia and tried some leucopenia and trhrombocytopenia. Which of
Indonesian traditional food. The onset of the the following virus is most likely responsible
disease is abrupt with abdominal cramps and for this infection?
watery diarrhea. She had no fever or nausea or f. Morbili
vomiting. The symptoms have resolved within 24 g. Dengue
hour and no subsequent recurrences. They report h. Influenza
the disease to district public health office. The i. Coxsackie
investigation found that one of the food products j. Rhinovirus
eaten by this tourist was contaminated by
suspected pathogens. 43. A 34-year-old woman complained a sudden
onset of high fever for 3 days with nausea,
39. What is the suspected pathogen may cause vomiting, headache, muscle ache. There was
the disease above? petechie on examination. The laboratory test
k. Salmonella typhi revealed leucopenia, thrombocytopenia, and
l. Shigella dysenteriae elevated of hematocrit. The patient most
m. Enterohemorragic E. coli likely acquired this infection by which of the
n. Staphylococcus aureus following modes?
o. Enterotoxigenic E.coli a. Intravenous drug use
b. Inhalation of contaminated dust
c. Close contact with a symptomatic patient
d. Bitten by an arthropods
40. Which of the following is the natural e. Eating some contaminated food
prevention for the disease above?
a. Bi Subsalicylate
b. Antibiotic
c. Probiotic
d. Chlorination
e. Hand hygiene

41. Approximately 4 hour after eating a meal in


restaurant, 3 members of a tourisms group Question 44-46 are linked to the following case:
develop a sudden onset of nausea, vomiting,
severe abdominal cramps, and diarrhea. A 20-year-old woman came to the emergency
Nobody got febrile. Which of the following room with high fever from 3 days ago. She also
vaccination is should be given to the traveler? has headache and pain in retroorbita, abdominal
f. Shigella vaccine pain and bloody stool. On physical examination,
g. Salmonella vaccine she was in an agitated condition. Her BP 90/60
h. Enterobacter vaccine mmHg, heart rate 110x/ minute, RR 20x/ minute
i. H Pylori vaccine and there was red spot in her arm, hand and leg.
44. Which of the following examination is the
most appropriate to diagnose this infection? 48. A 28 year old lady came to the Posyandu to
f. Prick test have a nutrition consultation. Last year, she
g. Tourniquet test delivered a stillbirth baby with a spinal
h. Complete blood test disorder. She and her husband start planning
i. Widal test to have a baby again. She asked for
j. Urine test suggestion about her dietary intake before she
got pregnant.
45. The patient was give IV line, and blood What kind of diet does she need to prevent
examination was done. Which of the the same disorder for her future baby?
following result is most appropriate with the f. Dark green leafy vegetables
patient’s diagnosis? g. Pasteurized milk
f. Trombositosis h. Deep water fishes
g. Leukocytosis i. Boiled eggs
h. Erythrositosis j. Lean beef
i. Elevated of hematokrit
j. Elevated of LED 49. A 1 year old girl came to the Puskesmas with
swollen gum for a week. From the history
46. In the forth day, the patient developed taking it was found that she was born normal
epistaksis, melena and more clear with normal birth weight, had been breastfed
hemorrhage in skin. Which of the following up to now and had a balanced diet with high
is responsible for his condition? intake of cooked vegetable and fruit since 6
a. Elevated of thrombocyte increasing months old. She had no history of chronic
coagulation diseases. Her other physical and laboratory
b. Fibrin degradation make dysfunction of examination was normal.
thrombocyte Based on the information given above, what
c. Elevated of kinin in circulation is the most likely cause of her complaint?
d. Plasmin elevated coagulation factor f. Protein deficiency due to prolonged
e. Complemen activation by plasmin breastfeeding
g. Carbohydrate deficiency due to increased
dr. DWI calorie need
h. Fat soluble vitamin deficiency due to
47. A 48 year old lady came to the outpatient high vegetable/fruit intake
clinic with chief complaint of fatigue and i. Water soluble vitamin deficiency due to
difficulty in concentration for the last 2 cooking process
weeks. The history taking revealed that she j. Mineral deficiency due to competition
had a balanced diet, no history of chronic with high vitamin absorption
disease and had been a heavy smoker for 20
years. On physical exam it was found that she dr. JULIANA
had anaemic conjunctiva with other findings (TUTOR)
was within normal limit. The laboratory
examination showed that she had haemolytic Question 50-51 are linked to the following case:
anaemia.
Based on the information given above, what A 9 year old girl came to the emergency room
is the most likely cause for her condition? with difficult of breathing. She had got high fever
f. Vitamin K deficiency since 3 days ago. Her temperature was up to
g. Copper deficiency 40oC. She also complained of rhinnorhea, cough,
h. Vitamin E deficiency and myalgia. On examination, you found
i. Iron deficiency increased tactile fremitus on the right side of the
j. Folate deficiency
lung, dullness on percussion. Laboratory result g. Gray syndrome
indicated leucopenia. h. Down syndrome
i. ADHD
50. Which criteria of avian influenza is the most j. Black water fever
appropriate with this patient’s condition?
a. Probable 54. A 24 year old man, recently returned from
b. Suspect Myanmar, presented very unwell febrile. His
c. Confirmed consciousness was GCS of 7. His thin blood
d. Definitive smear showed malaria falciparum. Which of
e. Absolute this following is the best treatment for this
patient?
a. Chloroquine
b. Mefloquine
c. Fansidar
d. Doxycycline
e. Quinine IV

51. Based on its ability to cause disease, avian


influenza is broadly divided into highly
pathogenic (HPAI) and low pathogenic
(LPAI) strains. Which subtypes are known to 55. A 35 year old man, a businessman, came to
cause the HPAI? private doctor. He asked the physician about
a. H1 and H3 his plan to go to Africa, which is endemic
b. H2 and H4 malaria. The physician suggested him to take
c. H5 and H7 prophylactic drug. Which medication (WHO
d. H6 and H8 recommended) would be given to this patient
e. H9 and H11 for prophylactic?
f. Kina
52. A 50 year old woman presented to clinic with g. Chloroquine
fluctuated fever (also called as step ladder h. Doxycycline
fever) since 7 days ago. On physical i. Kuinin
examination, you found “rose spots” on j. Primakuin
upper abdomen. What is the possible cause of
rose spots on this woman? 56. A 40 year old man came to hospital with four
a. Eritrocytopenia times generalized tonic-clonic seizure since
b. Leucopenia two week ago. There was no history of
c. Thrombocytopenia seizure and head trauma. A one year ago, he
d. Limfositosis had business in Papua for six months. During
e. Deficiency of clotting factor his stay in there, he often consumed under-
cooked pork. Brain MRI showed a small
53. A 27 year old woman in pregnancy had cystic with invaginated scolex inside. What
fluctuated fever since 2 weeks ago. She of this following is the most likely inside
looked pale, generalized weakness, and cyst?
headache. She went to public health center f. Eggs
and was prescribed oral chloramphenicol in g. Cycticercosis cellulose
doses of 500 mg four times daily and h. Cysticercosis bovis
paracetamol 500 mg three times daily. She i. Taenia solium
did not tell to the physician that she was j. Taenia saginata
pregnant. What is the side effect of
chloramphenicol on this patient? 57. A 15 year old girl of African origin was
f. Reye syndrome admitted with a history of headaches and a
generalised tonic seizure. Her clinical g. Botulism
examination was normal. Within a few hours h. Generalized seizure
of her admission she was found dead in her i. Tetanus
bed during the ward round. After autopsy, the j. Viral encephalitis
diagnosis was neurocycticercosis. Which of
this following brain’s location is the possible Question 60-61 are linked to the following case:
cause of patient’s sudden death?
a. The frontal lobe 60. 4. A worried mother brings her 18-month-old
b. The temporal lobe son to the emergency department because of
c. The parietal lobe a rash that developed today. She reports that
d. The lateral ventricle he has had a runny nose, conjunctivitis, and
e. The fourth ventricle diarrhea. On physical examination, he
appears toxic, is temperature to 104ºF (40°C)
dr. ROBERT SOETANDIO for the last 3 days and has diffuse 2- to 3-mm
erythematous rush beginning from their face.
58. You are precepting a resident who has just Of the following, the MOST likely cause of
evaluated a 4-year-old incompletely the rash is
immunized immigrant boy who has classic f. Adenovirus
varicella lesions and a history that is g. Coxsackievirus
consistent with this diagnosis. Of the h. Measles
following, the MOST accurate statement is i. Human herpesvirus 6
that j. Parvovirus B19
a. Lesions of both varicella and smallpox
follow a 7- to 10-day course from 61. Case no 4, if the son got worse. What would
eruption to resolution the complication happened?
b. Lesions of both varicella and smallpox f. Osteomylitis
frequently produce deep, pitted scars g. Encephalitis
c. Varicella lesions appear in stages or h. Otitis externa
crops; smallpox lesions are uniformly in i. Sinusitis
the same Stage of development j. Oral thrush
d. Varicella lesions are concentrated on the
face; smallpox lesions are concentrated Question 62-63 are linked to the following case:
over bony prominences
e. Varicella lesions are transient vesicles; 62. A 10-year-old boy who has moved to your
smallpox lesions are persistent pustules practice recently has sore throats, get high
until resolution of the illness temperature. The child reports a runny nose,
59. You are working in a refugee camp when a mild cough, and abdominal pain. Findings on
mother brings in her 8-day-old boy. The physical examination include a temperature
mother states he started becoming irritable 2 of 100.3°F (38°C), and vesicular lesions on
days ago, and now any loud noise appears to the soft palate. There is no cervical
cause him pain, as evidenced by muscle adenopathy or rash.
tightening and back arching causing his head Of the following, the MOST likely diagnosis
to nearly touch his feet. Physical examination is
reveals only a dried packing on his umbilical f. Adenovirus infection
cord, as is the local custom. He appears g. Coxsackie virus infection
normal until he is stimulated by touch or a h. Mononucleosis
loud noise, and then he begins to cry, stiffens, i. Sinusitis
and arches his back. The stiffness continues j. Streptococcal pharyngitis
until he calms down. Of the following, the
MOST likely diagnosis is
f. Bacterial meningitis
63. What is the name disease what is caused by He is difficult to get inspiratory and usually
coxsackie virus and the location in the finished by vomit. The laboratory results are
mouth? leucocytocis and absolute lymphocytes. Of
a. HFMD the following, the MOST likely diagnosis is
b. Herpangina f. Tonsilitis Folikularis
c. Conjungtivitis g. Bronchiolitis
d. Pleurodynia h. Pertussis
e. Streptococcal pharyngitis i. Laryngitis Akut
j. Faucial Diphteria
Question 64-65 are linked to the following case:

64. A 10-year-old girl presents with a history of


sore throat and difficulty breathing of 1 day’s 68. The boy in case 67 often gets secondary
duration. She returned 2 days ago from a trip infection. What the most organisms is usually
to Odessa (Ukraine). On examination she is cause the secondary infection?
very ill-appearing. She has some inspiratory a. Stafilokokus aureus
stridor and thick white-gray material b. Treponema palidum
covering her tonsils and faucial pillars, and c. Haemophilus Influenza
she has swelling of her neck, no d. Serratia marcescens
splenomegaly. The lymphocyte is normal. e. Corynebacterium diphteriae
What is the likely diagnosis?
f. Tonsilitis Folikularis Question 69-71 are linked to the following case:
g. Angina Plaut Vincent
h. Mononucleosis infectiosa 69. A baby is born with a rash identical.
i. Laryngitis Akut Questioning reveals that the mother had a
j. Faucial Diphteria febrile illness during the second trimester of
pregnancy. Examination reveals diffuse
65. Case no 64. if we want to give her DAT. raised purple skin lesions. There is no pallor,
How much should we give DAT? jaundice, or cyanosis. The eyes are cataract,
a. DAT 80.000 IU the heart has a 3/6 systolic murmur, and there
b. DAT 50.000 IU is enlargement of both the liver and the
c. DAT 100.000 IU spleen. There is no lymphadenopathy.
d. DAT 40.000 IU Of the following, the MOST likely diagnosis
e. DAT 60.000 IU is?
f. Congenital Toxoplasmosis
66. According IDAI schedule 2008, when is the g. Congenital Rubella
child getting the vaccination booster for the h. Congenital Cytomegalovirus
disease above after 18 months? i. Congenital Malaria
a. 5 years j. Congenital Varicella
b. 4 years
c. 3 years 70. What is vaccination which prevents disease
d. 12 years in number 69?
e. 15 years a. Measles
b. Varicella
Question 67-68 are linked to the following case: c. MMR
d. HPV
67. A 16-year-old boy presents with an annoying e. PCV
cough, which he has had for about 3 weeks.
The illness started with a runny nose. On 71. How old is the baby getting the vaccination?
examination he is completely normal, but he a. 15 months
exhibits several episodes of severe coughing. b. 16 months
c. 17 months f. Pausibacillar leprosy patient sensitive to
d. 18 months treatment
e. 19 months g. Multibacillar leprosy patient sensitive to
treatment
h. Multibacillar leprosy patient resistent to
dr. HANNA treatment
Question 72-73 are linked to the following case: i. Pausibacillar leprosy patient resistent to
treatment
A 45-year-old man was seen in the dermatology j. Pausibacillar leprosy patient prone to
clinic because of some nodules on his arm and nerve damage
body and sometimes feel pain with pressure. On
examinations we found erythematous nodules, 76. A 40-jear-old woman presented with a raised
smooth and shiny, diffuse infiltrate with ill define redness plaque resemble a doughnut with a
border, no fluctuation or erosion. There were pain raised border that she already had for 1 year,
on his elbow with nerve enlaergement. It was no itchy, no pain, not easily bleeding.
Sometime she feel weakness on her leg and
72. What is the most likely diagnosis? her sandal left behind while walking. On her
a. Yaws legs she developed an anular erythematous
b. Leprosy sharp border lesion, no scally, shiny surface
c. Anthrax and enlargement of the popliteal nerve. What
d. Cellulitis is the most likely diagnosis?
e. Carbuncle a. Leprosy
b. Yaws
73. What laboratory test would you do to confirm c. Celulitis
the clinical diagnosis? d. Anthrax
f. TPHA and VDRL
g. Gram preparation e. Insect bite
h. KOH preparation
77. A 30-year-old farmer complaned about
i. Skin scrapping weakness on his three fingers that he can not
j. Skin slit smear hold a plough He found some white patches
on his back, tummy arms, he felt numbness
74. The 21-year-old man was seen in on his right hand but not itching After one
dermatology clinic because of sore and year on medication he experience tenderness
redness on his previous skin lesion. He was and sore on his right elbow and the skin
on M D T treatment since 1.5 year ago. On patches became red. What is the most likely
skin examination they found some painfull diagnosis?
erythematous nodule spread over his body a. Framboesia
especially on arm and legs also fever. What b. Scorpion bites
is the most likely diagnosis ?
a. Celulitis c. Reversal reaction
b. Furuncle d. Lepromatous leprosy
c. Erythema nodosum e. Ramsay Hunt syndrome
d. Multibacillar leprosy
78. A patient on MDT treatment complaining
e. Pausibacillar leprosy about red urine that he experinced on the first
day of treatment. What is the suggestion for
75. A laboratory examination of a leprosy patient this condition ?
after 4 months M D T therapy revealed a. Stop Clofazimin, can cause pink
Bacterial index 6+ and morphological index brownish discolouration
50%. What is your conclution for this result ?
b. Continue the medication, its not a drug infection with the pork tapeworm, Taenia
adverse effect solium, because
c. Stop the medication because of a. Acute intestinal stoppage is less common
nefrotoxicity in beef tapeworm infection
d. Stop Dapsone can cause anemia b. Larval invasion does not occur in beef
hemolytic tapeworm infection
e. Change the treatment to single drug c. Toxic by-products are not given off by the
therapy adult beef tapeworm
d. The adult beef tapeworms are smaller
79. A 50-year-old man complained about the e. Beef tapeworm eggs cause less irritation of
treatment for his leprosy since it takes one the mucosa of the digestive tract
year medication with multiple drugs (MDT)
every day. Can you give some explaination 84. A woman, recently returned from Lampung,
about the length of treatment ? complains of having paroxysmal attacks of
a. There is no cure for leprosy chills, fever, and sweating; these attacks last
b. Leprosy bacteria can’t be killed by 1 drug a day or two at a time and recur every 36 to
c. M D T treatment can cure the disability 48 h. Examination of a stained blood
specimen reveals ringlike and crescent-like
d. To prevent from drug resistancy forms within red blood cells. The infecting
e. Due to his low immunity organism most likely is
f. Plasmodium falciparum
dr. KRISTO g. Plasmodium vivax
h. Plasmodium ovale
80. Tuberculosis become one of reemerging i. Plasmodium malarie
infectious disease because j. Plasmodium brasilianumi
a. More cases of tuberculosis found in
developing countries 85. Malaria is a significant worldwide public
b. Found more cases in childrek health problem. Which one of the following
c. Found more cases of lung cancer control methods for malaria is currently
d. Found more cases of drug resistance effective?
e. More people who smoke a. A vaccine
b. Chemoprophylaxis
c. Antibiotics
81. Antigenic shift of influenza viruses occurred d. White clothing
in e. Tick repellents
a. Haemagglutinin
b. Neuramidase 86. A man came to the emergency room with
c. M2 protein complaints body felt weak, fever, and
d. NS1 diarrhea since 7 days ago. Prior to the
e. NS2 emergency room, the patient had been to the
clinic 24 hours and given paracetamol and
82. Epilepsy which occurs in neurosistiserkosis cloramfenicol for 5 days, but the symptoms
caused by the did not subside. For definitive diagnosis in
a. Colloidal cyst patients with typhoid fever was the doctor
b. Vesicular cyst taking the material for culture from?
c. Calcification cyst a. blood
d. Necrosis cyst b. urine
e. Granuloma cyst c. stool
d. intestinal secretion
83. Human infection with the beef tapeworm, e. bone marrow
Taenia saginata, usually is less serious than
87. Typhoid fever pathogenesis is not affected by ×/min, RR 20x/min, temp. 39.1°C. Tongue is
a. Antigen Vi coated and tremor. Fetal USG shows normal.
b. Antigen H What is the most likely etiologic factor
c. Gastric acidity above?
d. Amount of bacteria f. Salmonella typhi
e. Antacid drug g. Plasmodium falciparum
h. Vibrio cholera
88. Amantadine, a synthetic antiviral agent used i. Hepatitis A
prophylactically againt influenza, is thought j. Escherichia coli
to act by
a. preventing production of viral capsid 92. A young man was admitted to hospital after
protein increasing left arm pain and paresthesia.
b. preventing virion release Several days ago, he contacted with a
c. preventing penetration of the virus into dog……. His symptoms increased and were
the host cell accompanied by hand spasm and sweating on
d. preventing uncoating of viral DNA the right side of the face and trunk. This
e. causing lysis of infected host cells by patient was admitted to the hospital the day
release of intracellular lysosomal after developing dysphagia, hypersalivation,
enzymes agitation, and generalized muscle twitching.
Which of the following could rapidly destroy
89. A 27-year-old female has just returned from a the pathogenic agent of this disease?
trip to Southeast Asia. In the past 24 hours, a. Infrared radiation
she has developed shaking, chills, and a b. Catalase
temperature 40o C. A blood smear reveals c. Alkali water
Plasmodium vivax. Which of the following d. Sunlight
agents should be used to eradicate the e. Heating at 60ºC for 30 minute
extraerythrocytic phase of the organism?
f. Primaquine 93. A 35-year-old man as a farm worker who was
g. Pyrimethamine working with pesticides is brought to the
h. Quinacrine emergency room with headache, vomiting,
i. Chloroquine salivation, diarrhea, muscle fasciculation,
j. Chologuanide difficulty walking, and difficulty speaking.
His clothing has been removed, he has been
90. The mechanism of action of chloroquine in washed, and he has been given activated
Plasmodium falciparum malaria is charcoal. What is the most effective
elimination of? remaining treatment for this case of pesticide
a. Secondary tissue schizons poisoning?
b. Exoerythrocytic schizoons f. Epinephrine
c. Erythrocytic stage g. Antacid
d. Asexual forms h. Spironolactone
e. Sporozoites i. Atropine
j. Hidrochlorothiazide
91. A 26-year-old nulliparous woman, in her
seventh month of pregnancy complains of a 7 94. A 27-year-old man came to a clinic with high
days history of fever especially at night. As fever for two days which is started with a
she recall, 2 weeks ago she ate gado-gado in chill. He also complained of swellings in his
coastal area when she out for duty for several left inguinal area. They were firm, discrete,
days. She also had diarrhea and abdominal and somewhat tender. His scrotums are also
discomfort. On physical examination reveals enlarged and tender. His doctor performed a
blood pressure 110/70 mmHg, heart rate 60 thick blood smear examination using blood
that was taken at night but found nothing.
What should the doctor do to make the c. Lactodectus mactan
diagnosis? d. Loxoxceles laeta
a. Biopsy of the scrotum e. Lytta vesicatoria
b. USG of the scrotum
c. CT scan of the scrotum 99. After contacting with hairy caterpillars, a
d. MRI of the scrotum farmer felt itching all over the whole body
e. Aspiration of the scrotum fluid and his eyes are swollen. The possible cause
of this disorder is:
a. Erucism
95. A young man presented to his general b. Lepidopterism
practitioner with a history of headache and c. Tick paralysis
fever together with a dry cough and muscle d. Arachnidism
ache. His nose was also blocked and his eyes e. Delusional parasitosis
watery.
What is your clinical diagnosis?
a. Influenza virus infection
b. Human herpes virus
c. Cytomegalovirus
d. Epstein-Barr virus Question 89-90 are linked to the following case:
e. Adenovirus
A 23-year-old man presents with extreme
96. A 4-year-old child presents with fever, swelling of his legs and scrotum. The skin
cough, conjunctivitis, coryza, photophobia, associated with the swollen areas is thick and
posterior cervical adenopathy. Red lesions scaly. The patient admits to an episode of fever
with a white center are present on the buccal associated with enlarged inguinal lymph nodes
mucosa. A generalized erythematous rash is some time ago, but did not think much of it.
also noted. What is the appropriate
management of this patient above? 100. Elephantiasis is caused by the obstruction
a. Immune globuline of :
b. MMR vaccine administration a. The arteries by microfilariae
c. Measurement of IgG antibody serum b. The arteries by adult worm
d. Measurement of IgM antibody serum c. The lymphatics by microfilariae
e. Antibiotics d. The chronic lymphatics by adult worm
e. Fibrosis of the lymphatic
97. To know the endemicity of malaria on Pulau
Seribu, spleen examination on children aged 101. Occult filariasis is Lymphatic filariasis
2-5 years was done. The result shows that based upon :
this place is mesoendemicity. a. Infection
What is the meaning of mesoendemicity? b. Allergy
a. Spleen rates not exceeding 10 % c. Infestation
b. Spleen rates between 11% and 50% d. Colonization
c. Spleen rates constantly over 50% e. Genetic
d. Spleen rates constantly over 75%
e. Parasite rate not exceeding10% 102. A patient complains of having nail size
excrement from his anus. On the stool
98. The cattle at animal husbandry are examination, parasite eggs contains a hexacanth
occasionally was dead because of tick embryo with six hooklets surrounded by radially
paralysis, the species of arthropod that can be striated spherical shell, 30 to 40 u in diameter
the cause to this disorder is; were found, the most likely diagnosis of this
a. Rhipicephalus sanguineus patient is :
b. Dermacentos andersoni a. Saginata taeniasis
b. Solium taeniasis c. Human erythrocyte
c. Cysticercosis d. Mosquito body
d. Sparganosis e. Mosquito proboscis
e. Dipylidiasis
107. Plasmodium falciparum is the most
Question 92-93 are linked to the following case: dangerous species because :
f. It make long term relaps
A 40-year-old man came to the clinic with g. Intermittent fever
complaints of having fever more than one-week, h. Gametocyte formation take place in
nausea, and fatigue. He came back from Timika, visceral organ
Papua. On blood smear examination, there are late i. Can make capillary obstruction in Central
trophozoites forms, schizonts with 12-24 Nervous System by parasite contain
merozoites, and the infected red cell is enlarged. erythrocyte
j. There are many drug resistance against P.
103. Of the following, the most likely caused falciparum
of parasite species is :
f. Plasmodium vivax Question 97-98 are linked to the following case:
g. Plasmodium falciparum
h. Plasmodium malariae A 30-years-old woman presents abdominal pain
i. Plasmodium ovale and diarrhea for 3 days duration. She does not
j. Plasmodium cynomolgi complain of nausea, vomiting, or fever. She has no
sick contacts or significant travel history. A stool
104. Fever symptom on the previous question sample is obtained, which reveals larvae form of of
referred to : Strongyloides stercoralis.
f. Sporozoites is in the blood
g. Hypnozoites in the liver is formed 108. In condition below adult form of
h. Merozoites discharged from schizonts Strongyloides stercoralis can be found in whole
enter the blood digestive tract and its larvae can be found in
i. The process of gametocytes in the blood visceral organ (lung, liver, gall bladder) is :
j. Pigment in the parasite is formed a. Eosinofilia
b. Hyperinfection
105. A 27 year old women suffering fever for 2 c. Retrofection
day. She brought to emergency department d. Hypereosinofilia
unconsciously and she had general seizure 2 e. Autoinfection
hour ago. On physical examination she is
comatous, GCS (Glasgow Coma Scale) : 3; BP
90/60 mmHg; t 41 °C. Lab. finding: Hb 10, 3 gr 109. Refer to the case above; further questioning
%; WBC 5800/mm3; platelet 39.000/mm3; reveals that the woman frequently gardens in
there is schizon form on blood smear. The her backyard. Of the following, which one is
most likely diagnosis is : the transmitted form?
a. Malaria viviax a. Fecal – anal transmission
b. Malaria falciparum b. Fecal – oral transmission
c. Malaria malariae c. Fecal – cutaneous transmission
d. Malaria ovale d. Direct contact with skin scales
e. Mix infection of malaria vivax and malaria e. Intravenous drug abuse
falciparum
110. A 10-month-old healthy male infant
106. Sexual life cycle of malaria parasite take travelling with his family to Africa was
place in : exposed 4 days ago to his 4-year-old native
a. Human liver cell African cousin who was ill at the time with
b. Human spleen cell
fever, cough, coryza and conjunctivitis. The c. Fecal-oral
baby was at risk to get infected by: d. Close contact
a. Corynebacterium diphtheriae. e. Animal bite
b. Haemophilus influenzae 114. A 4-year-old child presents with fever,
c. Bordetella pertussis conjunctivitis, photophobia, posterior
d. Morbilivirus cervical adenopathy, and coryza. Red lesions
e. Coxsackievirus with a white center are present on the buccal
mucosa. A generalized erythematous rash is
111. Her mother makes an overseas phone call also noted. What is the most likely diagnosis?
to your office asking your advice. a. Rubella
Appropriate management of this patient b. Kawasaki disease
includes: c. Adenovirus infection
a. Immune globuline d. Rubeola
b. MMR vaccine administration e. Varicella
c. Measurement of IgG antibody serum
d. Measurement of IgM antibody serum
e. Antibiotics TROPMED 1

112. A 3 year old girl came to pediatric 1. 42yo woman ocme to clinic noted white caps
outpatient clinic with her parents. Her mother in his stools. On stool exam parasit eggs contains
complaint that she had watery diarrhea since hexocant were found. Most likely source:
3 days. This was her 3rd episode of diarrhea in
A. Undercooked pork
the last 2 months. Her mother also noticed
that she was delayed in growth and B. Undercooked beef
development compared to her siblings. She
hardly gained weight and height since last C. Ingestion of larvae on the organism
year. The physical examination showed that contaminated food
her weight and height was below 5th
percentile for girls of her age. D. Ingestion of cystic
What is the most likely underlying
mechanism that caused her repeated episodes E. Ingestion of egg
of diarrhea?
a. Protein catabolism
b. Reduced food intake 2. 40 yo man, fever epistaxis nausea since 2days,
c. Altered lipid metabolism temp 39. Palpable liver. Rumple leed test+. Dhf!
d. Reduced bactericidal activities m confirming lab?
113. A 65-year-old-woman suddenly had A. Ns1
paralysis of her left leg. A few days ago, she
had headache, fever, sore throat, and nausea. B. Platelet count
She lives with her son; daughter in law and a
young grandchild who’s just received a C. Dengie igm igg
routine oral vaccination. She is taking
D. Wbc count
immunosuppressant for her kidney’s
transplant. Her vital signs and cranial nerve E. Ht n eritrosit count
examination are normal. A head CT scan and
lumbar MRI are also normal.
What is the most likely transmission of her
infection? 3. Worried mother bring her 18month son, rash
a. Droplet develop todayn runny nose, conjunctivitis,
b. Air borne diarrhea, lethargy, irritable. Temp 40c for 3days.
Diffuse 2-3mm erythemateous rash beginning 11. IgG dan IgM (+) low avidity IgG.
from face. Sever complication? Management?

A. Sinusitis. A no spesific management

B. Oral thrush B antepartum ganciclovir

C. Osteomyleitis C fetal usg and amnioncentesis

D. Encephalitis D CMV immunoglobin

E. Otiitis externa

12. 32 yo jaundice suffesion conjunctivitis


trombositopenia hepatosplenomegali...
4. 24 yo woman, fever of unknown origin, persist
for 10days, + diarrhea and now become a HbAs Ag
constipated. Brasycardia spleenomegaly
neutropenia lymphatocytosis. Most likely sites of B
infection?
C Anti dengue
A. Ileum
D IgM
B. colon
E MAT for leptospirosis
C. Appendix

D. Caecum
13. A 25 year old man is admitted to the hospital
E. Duodenum for evaluating increasing fever of unknown origin
along with malaise, headache, constipation. Nggk
vaksin terus ke daerah rural. Ada bradikardi,
stepladder fever, hepatosplenomegali. Antibiotik
9. 28yo male fever productive cough green yg dikasih apa?
mucous 5 hari lalu elevated BP HR 128 RR 28
temp 39,5 diffuse ronchi di chest x ray, A. Isnazid
pneumonia
B. vancomycin
A. Sirs b. Sepsis c viremia d bakteremia
C. Clindamycin
e septic shock
D. Metronidazole

E. chloramphenicole
10. 27 yo cold. Unprotected sex. How hiv virus
transmitted?

A saliva b needle stick injury c injecting drugs by 14. Fever high especially at night. Headache and
own needle pain behind eyes. Dignosis

d exposure of intact skin w contaminated blood A. Severe dengue if he was feeling restless and
lethargy
e mostly from mother to infant in gestation

15. Irritable, loud noise gara2 pain karena muscle


tightening and back arching. Phisical examination b. antiviral & supportive therapy
reveals only a dried pcking on his umbilical cord,
as is the local custom appears normal until he's c. supportive therapy & antibiotic
stimulated by touch or loud noise
d. supportive therapy without antiviral
A. Botulism

B. tetanus
19. 45 thn cowo ada nodul di arm & body pain
C. Meningitis with pressure. Erythematous nodules,smooth, and
shiny. Diffuse infiltrate with ill define border,no
D. Encephalitis erosion. Pain in elbow with nerve enlargement.
Lab?
E. generalized seizure
a. slit skin smear

b. skin scrapping
16. Suka main sm puppy, eosinophilia, enlarged
liver c. KOH

A. Pruritus Ani d. VDRL

B. cholecystitis

C. Billiary cirrhosis 20. 67 thn cewe flu like syndrome. Fever 40


derajat,anorexia,headache,muscle ache. 3 days
D. Creeping eruption later cough with sputum. Chest x ray
parenchymal infiltrate. Specimen untuk pcr?
E. visceral larva migrans
a. sputum

b. rectal swab
17. 33 thn cowo with C3 HIV disease.
Fever,nausea,vomiting,hipotensi. Minum c. midstream urine
zidovudin,lamivudine (3TC),abacavir,indinavir.
Develop fatigue, dyspnea,malaise. Then he stop d. CSF
medication. PE: hb turun,ast normal,alt
normal,bilirubin 2 (indirect 1,4). Cause? e. nasopharyngeal aspirate

a. side effect 3TC

b. side effect abacavir 21. Ujungkulon, liver enlarge, on off fever ?

c. side effect indinavir a. Bloodsmear

d. side effect zidovudin b. blood culture

18. 7 thn cowo malaise,anorex,fever,headache. 6 22. Inguinal node swelling, bubo, pustule, ?
classmates kaya gt juga punya symptoms yg diagnosis?
sama. Ada skin lesions macule,papule,vesicle
(multiform). Treatment? a. Tularemia

a. antiviral & antitoxin b. Yaws


c. Pes

27. 15 years old, male, come to Puskesmas. Fever


since 3 days ago. High fever especially at night.
23. Muscle pain? Headache (+), retroorbital pain (+), nausea (+),
vomit (+). What is the diagnosis according to
a. Trichinella spiralis WHO 2009?

A. Severe dengue if he was feeling restless and


24. Leptospirosis tertular dr mana? lethargy

B. Dengue fever with warning sign if Torniquet


test (+)
25. 25 years old, male, Balinese. Generalized
seizure. CT scan revealed several calcified C. Dengue fever with warning sign if rash (+)
regions. Brain biopsy was performed and D. Dengue fever with warning sign if abdominal
revealed scolex with hooklets. Which parasite is pain (+) just once last night
the most likely the cause:
E. Dengue fever with warning sign if abdominal
A. Echinococcus granulosus pain (+) more than once last night
B. Trichinella spiralis

C. Taenia saginata 28.


D. Taenia solium

E. Toxocara cati 29. Cowo 30 tahun datang ke puskes dengan


keluhan demam, menggigil, sakit kepala sejak 3
hari. Dengan riwayat demam sejak pergi ke ujung
26. 32 years old, male, come to emergency kulon national park. Pemeriksaan untuk
departement. 1 Week progressive fever, now diagnosa?
become delirium. Jaundice (+), conjungtival
suffusion (+), hepatomegaly (+), Murphy's sign A. VDRL
(-), petechiae on both legs (+). Lab testing was B. HIV ELISA
performed:
C. Urine culture
Leukocyte: 18.000
D. Blood smear
Thrombocyte: 98.000
E. Blood culture
Creatinine: 4

What further examination needed to be done?


30.
A. HBs Ag

B. Widal test
31.
C. Anti-Dengue

D. IgM anti HAV


32.
E. MAT for leptospira
the ethiology of the disease?

33. 10 y/o had bitten by dog 3 days ago in the A. Escherichia coli
right foot, he was brought to the clinic and
received antibiotic, analgesic and wound B. Salmonella typhi
cleansing w/o incision. The bitten site is now
painful, swollen, red and his mother notice that C. Bordetella pertusis
the boy has fever now. What's possibly happen to D. Shigella dysentriae
the boy?
E. Campylobacter jejuni
A. Local reaction to trauma and need anti..

B. Abscess right foot must get injection of


antibiotic and wound incision 36. 35 y/o man emergency from dyspneu
cyanosis, hemoptisis, chest pain, high fever,
C. Encephalitis stage of rabies infection need to malaise, non productive cough for 1 week. He
be isolated and rabies immunoglobulin works as a sheep farmer and was treated for dark
D. Local tetanus infection inject anti tetanus black skin lesion. Cbc normal, blood + sputum
toxoid and tetanus immunoglobulin negative, hemorrhagic mediastitis. Which
virulence factors most likely involved :
E. Rabies infection of right foot inject rabies
immunoglobulin and local vaccination of the A. Exotoxin
wound B. Endotoxin

C. Alpha hemolytic

D. Lipopolysacharide
34. 40 y/o man complaint of tingling , itching of E. Antiphagotic factor
legs 30 minutes after swimming in Lindu lake,
the next day small papules develops followed by
blisters of legs. Which paracyte infected him?
37. 45 y.o man, he is going to Mecca for Haji.
A. Hymenoptera sp What is the vaccine that can be given?
Meningococcal vaccine.
B. Fasciola hepatica

C. Fasciolopsis busci
38. 38 y.o man, he is going to Papua for 2 weeks.
D. Demencator What is the prevention drug of malaria?
E. Schistosoma japonicum Doxycycline!

69. a national rescue tem who often travel to


remote areas in indonesia has on-off fever and
35. 28 y/o man hospitalized for fever more than 7 chills for 15 days. Arthralgia and headache. PE:
days, fever higher at noon, felt lower at morning, enlarged liver and spleen. He was brought to ER
fatigue, dizzy, discomfort in stomach, because suddenly had seizure and finally lost his
constipation, lost appetite. Examination reveals conciousness . what is the characteristic of
temp 38C, mild pain in abdominal palpated bp pathogen that might cause the symptom above?
100/70 Hr 70 BPM, tremor, coated tongue. what's
a. invasion of the RBC by parasite
b. release of parasite antigen to produce d. darkfield (treponema)
febrile reaction
e. gram stain
c. transmission by injection of
sporozoites 93. a 40 years old male patient came to the
emergency department due to fever and upper
d. digestion of HB in food vacuoles to right abdomen pain since 3 days ago. PE : Blood
produce pigment pressure 120/80 Heart rate 112 RR 28 Temp 39,5.
Tenderness right upper quadrat. Lab : WBC
e. adherence of infected RBC to small 17.000 Hb 13,5 Platelet 100.000 SGOT 75 SGPT
BV endothel 17 ureum 80 creatinine 3,3. Hs abdominal USG
shows liver abscess. His condition called….

a. sepsis
70. fever since 3 days. High fever all day long.
3rd day platelet :148.000. HT: 42. Treatment b. viremia
suggestion?
c. bacteremia
a. inpatient hospital and IV fluid drip
d. severe sepsis
b. inpatient puskes with oral fluid intake
e. anaphylactic schock
c. inpatient hospital with oral fluid intake
and IV drip when cant take orally

d.outpatient with oral fluid and balik the 94. Following test would be the most reliable for
next dad u/ cek platelet confirming the diagnosis in case above :

e. outparient with oral fluid intake and a. culture of blood


balik 3 hari lahi u/ cek platelet
b. complete blood count

c. nucleic acid based test


71. HT naik 20%. Digigit sama sapa?
d. detection of toxin in his serum
A fruit bat

b. anopheles
95. a 34 years old man comes to dermatology
c. aedes aegepty with hyperpigmentated macules that spread on
his face and his upper arm color lesion white and
d. close contact with infected person feels scaly when t touched. Sometimes he feels
itchy and more itchy when he sweat. Woods light
e. direct contact with infected animal (+). Lab : large, blunt, short hyphae and thick
wall budding spores (spageti and meatball )

72. multiple papilomatous skin lesion. Wart like a. candida albicans


srawberry. Best bay diagnosed? b. aspergillus flavus
a. tissue culture c. microsporon canis
b. giemsa d. Malassezia furfur
c. antitoxin detect e. Trichosporon belgelii
marks +, edema, no blister/necrosis. Wound,
tender + on palpation

a) snake toxinspread blood


81. 13 old boy, fever, shaking chill, general ill,
fatigue muscleached & pain. Playing at garden. b) venom -> paralytic muscle
Stung by an insect in dorsal lower right arm. Skin
inflamed, sore, rash, worse in 1st 24hours. c) local inflammation response bite = midvenom
Erythematous surround papulafolicar with pus.
d) envenomation that cause necrotic tissue ->
a. Trepanoma pallidum pain, edema, abscess formation in right calf
b. H ducreyi
c. M. Leprae
d. S. Pyogenes 66. almond smell (intoxication)
e. C minitussismum
a) atropine
82. Rose spot, liver spleen enlarge, drave – b) naloxone
konstipasi, fever 10d, ill, bradikardi. Gold
standard nya? c) physostigmine

d) Na thiosulfate

83. A patient come with an itchy recured fungal


infection on the feet and need some advice to be
cured and some prevention measures so the 67. fever > 1 week, eritrosit winth ringforms and
diasease will not occure again, doctor already enlarged RBC
gave him some medication for fungal infection.
a) ovale
What kind of advise can make him healthy?
b) vivax
f. Don’t eat seafood
g. Use always sandals c) malariae
h. Keep the skin dry and clean
i. Don’t use shoes all day long d)knowlesi
j. Don’t scratch, will spread the infection
e) falciparum

84. 27 y.o pregnant woman found HIV+ on 1. An-8-year-old girl sustains a large
prenatal blood testing, CD4T 410, viral load laceration contaminated with dirt after
35000. Ga mau antriretroviral. To decrease risk falling from her bike. Her mother can’t
nyebar ke anak? recall how many doses of tetanus toxoid
her daughter has received. Management
a. Do nothing of tetanus prophylaxis in this situation of
b. Cesar unknown history of prior doses of tetanus
c. Prophylaxis HSV II toxoid includes:
d. Retroviral after delivery A. Adult-type dT
e. Breast feed until 6 month B. TIG
C. dT and TIG
D. Haemophilus influenzae type b
65. 27 yo man. Snake bite in lower calf 3 hours conjugate vaccine containing tetanus
ago. Swollen, painful, snake: elapidae. Fang toxoid and TIG
E. DPT Answer: E

Answer: C
6. What laboratory test to confirm the
diagnosis in this patient?
2. If this patient developed seizures, what A. Blood culture
substance have responsible in this B. Throat swab
patient? C. Chest X-ray
A. Tetanospasmin D. Measurement of IgG antibody serum
B. Tetanolisin E. Complete blood count
C. Adenylate cyclase toxin
D. Fillamentous hemagglutinin Answer: B
E. Pertactin
Answer: A

3. A 10-month-old healthy male infant 7. A 2-year-old boy was brought to


travelling with his family to Africa was emergency unit Balaraja Hospital with
exposed 4 days ago to his 4-year-old difficulty in breathing, choking, gasping,
native African cousin who was ill at the and whoop cough.
time with fever, cough, coryza and What is your most likely diagnosis?
conjunctivitis. The baby was at risk to get A. Bordetella pertussis infection
infected by: B. Haemophilus influenzae infection
A. Corynebacterium diphtheriae. C. Coxsackievirus infection
B. Haemophilus influenzae D. Adenovirus infection
C. Bordetella pertussis E. Corynebacterium diphtheriae
D. Morbilivirus infection
E. Coxsackievirus Answer: A

Answer: D
8. In this case all the family members
4. Her mother makes an overseas phone call should get the prophylaxis antibiotics as
to your office asking your advice. follow:
Appropriate management of this patient A. Amoxicillin 30-50 mg/kg/day p.o. for
includes: 14 days
A. Immune globuline B. Amoxicillin 30-50 mg/kg/day p.o. for
B. MMR vaccine administration 7 days
C. Measurement of IgG antibody serum C. Erithromycin 40-50 mg/kg/day p.o.
D. Measurement of IgM antibody serum for 14 days
E. Antibiotics D. Erithromycin 40-50 mg/kg/day p.o.
Answer: A for 7 days
E. Tetrasiklin 25-50 mg/kg/day p.o. for
5. A 3-year-old boy was brought to 7 days
emergency unit Tangerang Hospital with
stridor, dyspnea, and “croupy” cough. Answer: C
What is your most likely diagnosis?
A. Bordetella pertussis infection
B. Haemophilus influenzae infection
C. Coxsackievirus infection
D. B. Parapertussis infection
E. Corynebacterium diphtheriae
infection

S-ar putea să vă placă și